+ All Categories
Home > Documents > SET 2 - TEST 19 POLITY EXPLANATION

SET 2 - TEST 19 POLITY EXPLANATION

Date post: 27-Feb-2022
Category:
Upload: others
View: 6 times
Download: 0 times
Share this document with a friend
98
PRESTORMING TM 2021 SET 2 - TEST 19 POLITY EXPLANATION Q.1) Consider the following statements: 1. Absence of external constraints on an individual. 2. Availability of adequate material resources for individual development. 3. Opportunity to participate in the decision making process. Which of the above is/are based on the principle of Positive liberty? (a) 1 only (b) 2 and 3 only (c) 1 and 2 only (d) 1,2 and 3 EXPLANATION Absence of external constraints is based on the principle of negative liberty. There should be fewer constraints made by the society and the government on the individual which is based on the positive liberty. So, Statement 1 is not correct. The freedom of the individual must not be constrained on the account of poverty and unemployment, so it necessary for the society and the government to make availability of adequate material resources for individual development which is based on positive liberty. So, Statement 2 is correct. People must have the opportunity in the decision making process, so that the laws made can reflect their choices, or at least take those preferences into account. So, Statement 3 is correct. ADDITIONAL INFORMATION Positive Liberty Positive liberty is concerned with explaining the idea of ‘freedom to’. They are in response to the answer ‘who governs me?’ to which the ideal answer is ‘I govern myself ’. Positive liberty discussions have a long tradition that can be traced to Rousseau, Hegel, Marx, Gandhi, Aurobindo, and also to those who draw their inspiration from these thinkers. It is concerned with looking at the conditions and nature of the relationship between the individual and society and of improving these conditions such that there are fewer constraints to the development of the individual personality.
Transcript

PRESTORMINGTM 2021

SET 2 - TEST 19 POLITY EXPLANATION

Q.1) Consider the following statements:

1. Absence of external constraints on an individual.

2. Availability of adequate material resources for individual development.

3. Opportunity to participate in the decision making process.

Which of the above is/are based on the principle of Positive liberty?

(a) 1 only

(b) 2 and 3 only

(c) 1 and 2 only

(d) 1,2 and 3

EXPLANATION

Absence of external constraints is based on the principle of negative liberty. There should be

fewer constraints made by the society and the government on the individual which is based on

the positive liberty. So, Statement 1 is not correct.

The freedom of the individual must not be constrained on the account of poverty and

unemployment, so it necessary for the society and the government to make availability of

adequate material resources for individual development which is based on positive liberty.

So, Statement 2 is correct.

People must have the opportunity in the decision making process, so that the laws made can

reflect their choices, or at least take those preferences into account. So, Statement 3 is

correct.

ADDITIONAL INFORMATION

Positive Liberty

• Positive liberty is concerned with explaining the idea of ‘freedom to’.

• They are in response to the answer ‘who governs me?’ to which the ideal answer is ‘I govern

myself ’.

• Positive liberty discussions have a long tradition that can be traced to Rousseau, Hegel, Marx,

Gandhi, Aurobindo, and also to those who draw their inspiration from these thinkers. It is

concerned with looking at the conditions and nature of the relationship between the individual

and society and of improving these conditions such that there are fewer constraints to the

development of the individual personality.

• The individual to develop his or her capability must get the benefit of enabling positive

conditions in material, political and social domains.

Negative Liberty

• ‘Negative liberty’ seeks to define and defend an area in which the individual would be

inviolable, in which he or she could ‘do, be or become’ whatever he or she wished to ‘do, be

or become’.

• This is an area in which no external authority can interfere.

• It is a minimum area that is sacred and in which whatever the individual does, is not to be

interfered with.

• The existence of the ‘minimum area of non-interference’ is the recognition that human

nature and human dignity need an area where the person can act unobstructed by others.

• Negative liberty arguments are in response to the question: ‘Over what area am I the

master?’ It is concerned with explaining the idea of ‘freedom from’.

Q.2) Consider the following statements:

1. The Concept of liberalism advocated by Indian Constitution prioritises individual rights over

those of society.

2. The liberalism practiced by thinkers like Swami Vivekanand introduced the spirit of social

justice within orthodox Hinduism.

Which of the above statements is//are correct?

(a) 1 only

(b) 2 only

(c) Both 1 and 2

(d) Neither 1 nor 2

EXPLANATION

Classical liberalism always privileges the rights of the individuals over demands of social

justice and community values. The liberalism of the Indian Constitution differs from this

version because Indian liberalism was always linked to social justice and not always

privileges the rights of the individuals over community values. So, Statement 1 is not

correct.

There are two Streams of Indian Liberalism. The First Stream began with Rammohun Roy.

It emphasized individual rights, particularly the rights of women. The second stream

included thinkers like K.C. Sen, Justice Ranade, and Swami Vivekananda. They introduced

the spirit of social justice within orthodox Hinduism. So, Statement 2 is correct.

Q.3) The structural part of Indian Constitution is derived from which of the following

sources?

(a) American Constitution

(b) British Constitution

(c) Government of India act of 1935

(d) None of these

EXPLANATION

The structural part of the Indian constitution is derived from the Government of India Act

1935. The Federal scheme, Judiciary, Governors, Emergency powers, the Public Service

Commission and most of the administrative details are drawn from this act. Government of

India Act 1935 was passed by British Parliament in August 1935.

The features of this Act were as follows:

1. It provided for the establishment of an All-India Federation consisting of provinces

and princely states as units. The Act divided the powers between the Centre and

units in terms of three lists Federal List (for Centre, with 59 items), Provincial List

(for provinces, with 54 items) and the Concurrent List (for both, with 36 items).

Residuary powers were given to the Viceroy. However, the federation never came into

being as the princely states did not join it.

2. It abolished dyarchy in the provinces and introduced ‘provincial autonomy’ in its place. The

provinces were allowed to act as autonomous units of administration in their defined

spheres. Moreover, the Act introduced responsible Governments in provinces, that is, the

Governor was required to act with the advice of ministers responsible to the provincial

legislature. This came into effect in 1937 and was discontinued in 1939.

3. It provided for the adoption of dyarchy at the Centre. Consequently, the federal

subjects were divided into reserved subjects and transferred subjects. However, this

provision of the Act did not come into operation at all.

4. It introduced bicameralism in six out of eleven provinces. Thus, the legislatures of

Bengal, Bombay, Madras, Bihar, Assam and the United Provinces were made

bicameral consisting of a legisilative council (upper house) and a legislative assembly

(lower house). However, many restrictions were placed on them.

5. It further extended the principle of communal representation by providing separate

electorates for depressed classes (Scheduled Castes), women and labour (workers).

6. It abolished the Council of India, established by the Government of India Act of 1858.

The secretary of state for India was provided with a team of advisors.

7. It extended franchise. About 10 per cent of the total population got the voting right.

8. It provided for the establishment of a Reserve Bank of India to control the currency

7and credit of the country.

9. It provided for the establishment of not only a Federal Public Service Commission,

but also a Provincial Public Service Commission and Joint Public Service Commission

for two or more provinces.

10. It provided for the establishment of a Federal Court, which was set up in 1937.

So, option (c) is correct.

ADDITIONAL INFORMATION

Others sources of Indian constitution

Constitution of USA Fundamental Rights

Federal Structure of Government

Electoral College

Independence of the Judiciary and separation of powers among the three branches of the Government

Judicial Review

President as the Supreme Commander of Armed Forces

Equal Protection Under Law

British Constitution Parliamentary Government

Single Citizenship

Rule of law

Writs

Speaker and his role

Law Making Procedure

Procedure established by Law

Irish Constitution Directive Principles of State Policy

Nomination of members to Rajya Sabha Method of Election of President

USSR Fundamental Duties under Article 51-A

A Constitutionally mandated Planning Commission

South Africa Procedure for Amendment

Election of Rajya Sabha members

Japan Procedure Established by Law

Q.4) Consider the following statements:

1. Collective responsibility of the executive to the legislature

2. Principle of Parliamentary sovereignty

3. Presence of an elected head of state

Which of the above are the essential features of a Parliamentary system of government?

(a) 1 and 2 only

(b) 1 only

(c) 2 and 3 only

(d) 1, 2 and 3

EXPLANATION

1. Collective responsibility – The executive are collectively responsible to the Parliament.

They act as a team and swim and sink together. The principle of collective

responsibility implies that Parliament can remove the ministry by passing a vote of no

confidence. So Collective responsibility of the executive to the legislature is the essential

feature of the Parliamentary system of government.

2. Principle of Parliamentary sovereignty - Parliamentary sovereignty is a principle of the

UK constitution. It makes Parliament the supreme legal authority in the UK, which can

create or end any law. Generally, the courts cannot overrule its legislation.

Parliamentary sovereignty is the most important feature of a Parliamentary form of

Government, but it is not an essential feature. For example, India follows parliamentary

form of government but Indian Parliament is not sovereign as Britain. So it is not the

essential feature.

3. Presence of an elected head of state – the presence of an elected head of the state may

not be the essential feature of the Parliamentary system of government. For example,

the Britain follows Parliamentary system of government where it does not contain

elected head of the state. So, option (b) is correct.

Q.5) Amongst the following, which is the most important function performed by a

Constitution?

(a) Constitution provides for a set of basic rules that allow for minimal coordination

amongst the members of the society.

(b) The Constitution provides for the procedure to constitute the government.

(c) The Constitution protects the rights of the people of a country by providing for a limited

government.

(d) The Constitution empowers the government to fulfill the aspirations of a society and create

conditions for a just society.

EXPLANATION

Every Society has people belonging to different groups such as occupation, religion, caste etc.

There would be conflict among these groups if there were no basic norms to govern them. So

the first and the most important function of the Constitution is to provide a set of basic rules

that allow for minimal coordination amongst the members of the society. So, option (a) is

correct.

Q.6) Consider the following statements:

1. The Indian Constitution did not enjoy public authority because the members of the Constitution

assembly were not directly elected by the people.

2. Two hundred ninety-nine members of the Constituent Assembly were present on 26 November

1949 and appended their signature to the Constitution as finally passed.

3. The Constitution assembly has twenty six members from the Scheduled caste community.

Which of the above statements is//are correct?

(a) 1 and 2 only

(b) 2 only

(c) 3 only

(d) 1 and 3 only

EXPLANATION

Some countries have subjected their constitution to a full-fledged referendum, where all

the people vote on the desirability of a constitution. The Indian constitution was never

subject to such a referendum but nevertheless it carried enormous public authority

because it had the consensus and backing of leaders who were themselves popular.

Although the constitution itself was not subjected to a referendum, the people adopted it

as their own by abiding by its provisions. So, Statement 1 is not correct.

The Constitution of India was adopted on 26 November, 1949 and the hon'ble members

appended their signatures to it on 24 January, 1950. Out of 299 members, 284 members

actually signed the Constitution. So, Statement 2 is not correct.

The Constitution assembly has 31 members from the Scheduled caste community. The

Constitution Assembly has 163 Hindus, 80 Muslims, 6 Indian Christians, 6 Backward

Tribes, 4 Sikhs, 3 Anglo Indians and 3 Parsis. So, Statement 3 is not correct.

ADDITIONAL INFORMATION

CONSTITUENT ASSEMBLY

• The Constituent Assembly took almost three years (two years, eleven months and seventeen

days to be precise) to complete its historic task of drafting the Constitution for Independent

India. During this period, it held eleven sessions covering a total of 165 days. Of these, 114

days were spent on the consideration of the Draft Constitution.

• As to its composition, members were chosen by indirect election by the members of the

Provincial Legislative Assemblies, according to the scheme recommended by the Cabinet

Mission.

• The arrangement was: (i) 292 members were elected through the Provincial Legislative

Assemblies; (ii) 93 members represented the Indian Princely States; and (iii) 4 members

represented the Chief Commissioners' Provinces.

• The total membership of the Assembly thus was to be 389. However, as a result of the partition

under the Mountbatten Plan of 3 June, 1947, a separate Constituent Assembly was set up for

Pakistan and representatives of some Provinces ceased to be members of the Assembly. As a

result, the membership of the Assembly was reduced to 299.

• On 13 December, 1946, Pandit Jawaharlal Nehru moved the Objectives Resolution.

• The Constitution of India came into force on 26 January, 1950. On that day, the Assembly

ceased to exist, transforming itself into the Provisional Parliament of India until a new

Parliament was constituted in1952

IMPORTANT COMMITTEES OF THE CONSTITUENT ASSEMBLY AND THEIR CHAIRMEN

Name of the Committee Chairman

Committee on the Rules of Procedure Rajendra Prasad

Steering Committee Rajendra Prasad

Finance and Staff Committee Rajendra Prasad

Credential Committee Alladi Krishnaswami Ayyar

House Committee B. Pattabhi Sitaramayya

Order of Business Committee K.M. Munsi

Ad hoc Committee on the National Flag Rajendra Prasad

Committee on the Functions of the Constituent

Assembly G.V. Mavalankar

States Committee Jawaharlal Nehru

Advisory Committee on Fundamental Rights,

Minorities and Tribal and Excluded Areas Vallabhbhai Patel

Minorities Sub-Committee H.C. Mookherjee

Fundamental Rights Sub-Committee J.B. Kripalani

North-East Frontier Tribal Areas and Assam Exluded

& Partially Excluded Areas Sub-Committee Gopinath Bardoloi

Excluded and Partially Excluded Areas (Other than

those in Assam) Sub-Committee A.V. Thakkar

Union Powers Committee Jawaharlal Nehru

Union Constitution Committee Jawaharlal Nehru

Drafting Committee B.R. Ambedkar

Q.7) With respect to the composition of the Constituent Assembly, consider the following

statements:

1. The seats allotted to the British provinces were divided among three communities- Muslims,

Sikhs and general, in proportion to their population.

2. The representatives of the princely states were to be nominated by the heads of the princely

states.

3. The representatives of the provinces were elected directly by the people of their respective

community.

Which of the above statements is//are correct?

(a) 1 and 2 only

(b) 2 and 3 only

(c) 1 and 3 only

(d) 1,2 and 3

EXPLANATION

Seats allocated to each British province were to be divided among the three principal

communities Muslims, Sikhs, and General (all except Muslims and Sikhs), in proportion to

their population. So, Statement 1 is correct.

The representatives of the princely states were to be nominated by the heads of the princely

states. So, Statement 2 is correct.

Members of each community (not directly elected by people) in the Provincial Legislative

Assembly elected their own representatives by the method of proportional representation with

the single transferable vote. The method of selection in the case of representatives of Princely

States was to be determined by consultation. Thus representatives of the provinces were not

elected directly by the people of their respective community. So, Statement 3 is not correct.

ADDITIONAL INFORMATION

COMPOSITION OF CONSTITUENT ASSEMBLY

• Each province and princely state (or group of states in case of small states) was to be allotted

seats in proportion to their respective population. Roughly, one seat was to be allotted for every

million population.

Allocation of seats in Constituent Assembly

S.NO Areas Seats

1. British Indian Province

292

2. Princely States 93

3. Chief Commissioners Province

4

Total 389

Q.8) Consider the following:

1. Indian Independence act, 1947

2. Government of India act, 1935

3. Privy Council Jurisdiction act

Which of the above acts was/were repealed with the Commencement of the Indian Constitution?

(a) 1 only

(b) 1 and 2 only

(c) 2 and 3 only

(d) 1,2 and 3

EXPLANATION

With the commencement of the Constitution, the Indian Independence Act of 1947 and the

Government of India Act of 1935, with all enactments amending or supplementing the latter

Act, were repealed. The Abolition of Privy Council Jurisdiction Act was however continued.

So, option (b) is correct.

ADDITIONAL INFORMATION

The Abolition of Privy Council Jurisdiction Act

• In 1949, the Abolition of Privy Council Jurisdiction Act was passed by the Indian Government.

• This Act accordingly abolished the jurisdiction of the Privy Council to entertain new appeals

and petitions as well as to dispose of any pending appeals and petitions.

• It also provided for the transfer of all cases filed before Privy Council to the Federal Court in

India. All powers of the Privy Council regarding appeals from the High Court were conferred to

the Federal Court.

Q.9) Consider the following statements:

1. The 42nd Constitutional amendment act of 1976 added a set of eleven fundamental duties

under part IV-A of the Constitution.

2. The 61st Constitutional amendment act reduced the voting age from 21 years to 18 years in

India.

3. The 86th Constitutional amendment act made elementary education a fundamental right in

India.

Which of the above statements is//are correct?

(a) 1 only

(b) 1 and 2 only

(c) 2 and 3 only

(d) 3 only

EXPLANATION

The 42nd Constitutional amendment act of 1976 added a set of 10 fundamental duties (not

eleven) under part IV-A of the Constitution. 86th Constitutional amendment act added

Eleventh fundamental duty to provide opportunities for education to his child or ward

between the age of six and fourteen years. So, Statement 1 is not correct.

Sixty-first Amendment Act, 1988 reduced the voting age from 21 years to 18 years in India.

So, Statement 2 is correct.

Article 21 A declares that the State shall provide free and compulsory education to all

children of the age of six to fourteen years in such manner as the State may determine. This

provision makes only elementary education a Fundamental Right and not higher or

professional education. This provision was added by the 86th Constitutional Amendment Act

of 2002. So, Statement 3 is correct.

Q.10) Consider the following:

“To protect and improve the environment and to safeguard forests and Wildlife.’’

The above provision is contained in which of the following parts of the Indian Constitution?

(a) Fundamental Rights

(b) Fundamental Duties

(c) Directive principles of State Policy(DPSP)

(d) None of these

EXPLANATION

Article 48 A in DPSP directs the state to protect and improve the environment and to

safeguard forests and Wildlife. So, option (c) is correct.

ADDITIONAL INFORMATION

DPSP

• The Directive Principles of State Policy are enumerated in Part IV of the Constitution from

Articles 36 to 51.

• The framers of the Constitution borrowed this idea from the Irish Constitution of 1937, which

had copied it from the Spanish Constitution.

• Dr. B.R. Ambedkar described these principles as ‘novel features’ of the Indian Constitution.

• The Directive Principles along with the Fundamental Rights contain the philosophy of the

Constitution and are the soul of the Constitution. Granville Austin has described the Directive

Principles and the Fundamental Rights as the ‘Conscience of the Constitution”.

• DPSP are non-justifiable that is they are not legally enforceable by the courts for their

violation.

• The Constitution does not contain any classification of Directive Principles. However, on the

basis of their content and direction, they can be classified into three broad categories, viz,

socialistic, Gandhian and liberal-intellectual.

• DPSP aims at establishing social and economic democracy in the country.

• They require legislation for their implementation. Hence they are not automatically enforced.

• The courts cannot declare a law violative of any of the Directive Principles as unconstitutional

and invalid. However, they can uphold the validity of a law on the ground that it was enacted

to give effect to a directive.

Q.11) Consider the following statements:

1. India is an independent, sovereign, socialist republic.

2. India shall be a Union of erstwhile British Indian territories, Indian States, and other parts

outside British India and the such Indian States as are willing to be a part of the Union.

3. All powers and authority of sovereign and independent India and its Constitution shall flow

from the people.

Which of the above were the part of historic Objective resolution?

(a) 1 and 2 only

(b) 2 and 3 only

(c) 1 and 3 only

(d) 1,2 and 3

EXPLANATION

On December 13, 1946, Jawaharlal Nehru moved the historic ‘Objectives Resolution’ in the

Assembly. It laid down the fundamentals and philosophy of the constitutional structure.

Following are the Objectives Resolution moved by Jawaharlal Nehru:

1. This Constituent Assembly declares its firm and solemn resolve to proclaim India

as the Independent Sovereign Republic( socialist is not mentioned in Objective

resolution) to draw up for her future governance a Constitution.

2. Wherein the territories that now comprise British India, the territories that now

form the Indian States and such other parts of India as are outside India and the

States as well as other territories as are willing to be constituted into independent

sovereign India, shall be a Union of them all; and

3. wherein the said territories, whether with their present boundaries or with such others

as may be determined by the Constituent Assembly and thereafter according to the law

of the Constitution, shall possess and retain the status of autonomous units together

with residuary powers and exercise all powers and functions of Government and

administration save and except such powers and functions as are vested in or assigned

to the Union or as are inherent or implied in the Union or resulting therefrom; and

4. wherein all power and authority of sovereign independent India, its constituent

parts and organs of Government are derived from the people and

5. wherein shall be guaranteed and secured to all the people of India justice, social,

economic and political; equality of status of opportunity, and before the law; freedom of

thought, expression, belief, faith, worship, vocation, association and action, subject to

law and public morality; and

6. wherein adequate safeguards shall be provided for minorities, backward and tribal areas,

and depressed and other backward classes; and

7. whereby shall be maintained the integrity of the territory of the Republic and its

sovereign rights on land, sea and air according to justice and the law of civilized nations;

and

8. This ancient land attains its rightful and honoured place in the world and makes its full

and willing contributions to the promotion of world peace and the welfare of mankind.

The Resolution was adopted by the Assembly on Jan 27 , 1947. It influenced the eventual

shaping of the Constitution through all its subsequent stages. Its modified version forms

the Preamble of the present Constitution.

Hence all three are the are the Objective Resolution passed by Nehru. So, option (d) is

correct.

Q.12) The Preamble secures to all the citizens equality of status and opportunity. Which of the

following provisions of the Constitution seek to give effect to the political dimension of

equality in the above context?

1. Prohibition of discrimination on grounds of religion, race, caste, sex or place of birth.

2. Elections to the Lok Sabha and state assemblies shall be on the basis of Adult suffrage.

3. State shall secure to men and women equal right to and adequate means of livelihood.

4. No person shall be declared ineligible for inclusion in electoral rolls on grounds of religion, race,

caste or sex.

Select the correct answer using the code given below:

(a) 1 and 2 only

(b) 2 and 4 only

(c) 2,3 and 4 only

(c) 1 and 3 only

EXPLANATION

There are two provisions in the Constitution that seek to achieve Political equality.

1. Article 325: No person shall be declared ineligible for inclusion in electoral rolls on

grounds of religion, race, caste, or sex.

2. Article 326: Elections to the Lok Sabha and state assemblies shall be on the basis of

Adult suffrage.

Prohibition of discrimination on grounds of religion, race, caste, sex or place of birth is not

political equality ( Article 15).

The state shall secure to men and women equal right to and adequate means of livelihood

and equal pay for equal work is a DPSP (Article 39).

So, option (b) is correct.

ADDITIONAL INFORMATION

PREAMBLE

• The Preamble to the Indian Constitution is based on the ‘Objectives Resolution’, drafted and

moved by Pandit Nehru, and adopted by the Constituent Assembly’. It has been amended by

the 42nd Constitutional Amendment Act (1976), which added three new words—Socialist,

Secular and Integrity.

TEXT OF THE PREAMBLE

The Preamble in its present form reads:

We, THE PEOPLE OF INDIA, having solemnly resolved to constitute India into a SOVEREIGN

SOCIALIST SECULAR DEMOCRATIC REPUBLIC and to secure to all its citizens:

• JUSTICE, Social, Economic and Political;

• LIBERTY of thought, expression, “belief, faith and worship;

• EQUALITY of status and of opportunity; and to promote among them all;

• FRATERNITY assuring the dignity of the individual and the unity and integrity of the Nation.

Q.13) Consider the following statements:

1. Preamble is a part of the Indian Constitution but it cannot be used to interpret the provisions

of the Constitution.

2. Preamble is neither the source of rights nor duties for the citizens.

3. Preamble can be amended only as per the provisions of article 368 of the Constitution.

Which of the above statements is//are correct?

(a) 1 and 2 only

(b) 2 only

(c) 2 and 3 only

(d) 1 and 3 only

EXPLANATION

A preamble is a part of the Indian Constitution. In the Kesavananda Bharati case, SC held

that the Preamble is part of the Indian Constitution. It is observed that Preamble is of extreme

importance and the Constitution should be read and interpreted in the light of the grand

and noble vision expressed in the Preamble. So, Statement 1 is not correct.

Preamble is neither a source of power to legislature nor a Prohibition upon the powers of

legislature. It is neither the source of rights nor the duties for the citizens. Preamble indicates

only the general purposes for which the people ordained and established the Constitution. It

cannot be regarded and has never been regarded as the source of any source of any

substantive power conferred on the Government or any of its departments. So, Statement 2 is

correct.

Preamble can be amended only under Article 368 of the Constitution. The SC held that

Preamble is a part of the Constitution and the ‘basic features’ cannot be altered by an

amendment under Article 368. So, Statement 3 is correct.

Q.14) Which of the above statements is incorrect?

(a) Socialism believes that all the resources should be owned by the state and used for people's

welfare.

(b) Socialism advocates the elimination of private property.

(c) Socialism empowers the state to use force to take away private property.

(d) All the above statements are correct.

EXPLANATION

Socialism's main motive is to abolishing of Private property. Socialists were against private

property. They saw it as the root of all social ills of the time. Therefore, the socialists wanted the

whole population to control the property rather than an individual so that more attention would

be paid to collective social interests.

Under socialism, states are also empowered to use force to take away private property.

Under socialism, all citizens share the economic resources equally as allocated by a

democratically-elected government. It is where all the resources are owned by the state and

distributed equally to the people's welfare. So, option (d) is correct.

ADDITIONAL INFORMATION

Socialism

• Socialism is an economic system where everyone in the society equally owns the factors of

production. The ownership is acquired through a democratically elected government. It could

also be a cooperative or public corporation in which everyone owns shares.

• The socialist government employs centralized planning to allocate resources based on both the

needs of individuals and society as a whole.

• Economic output is distributed according to each individual’s ability and level of contribution.

Difference between socialism and communism

• Under communism, the people are compensated or provided for based on their needs. In a

pure communist society, the government provides most or all food, clothing, housing and other

necessities based on what it considers to be the needs of the people. Socialism is based on the

premise the people will be compensated based on their level of individual contribution to the

economy. Effort and innovation are thus rewarded under socialism.

Q.15) With reference to India Secularism, consider the following statements:

1. The Indian Secularism envisions equal protection by state to all religions.

2. The right to freedom of religion of the citizens can be restricted on the grounds of public order,

health and morality.

3. The citizens cannot be forced to pay taxes for the promotion of any particular religion.

Which of the above statements is//are correct?

(a) 1 and 2 only

(b) 2 and 3 only

(c) 1 and 3 only

(d) 1, 2 and 3

EXPLANATION

The Indian Constitution embodies a positive concept of Secularism that all the religion in

our country irrespective of their strength has equal protection, same status, and support

from the State. So, statement 1 is correct.

Freedom of religion is subject to certain limitations. The government can impose

restrictions on the practice of freedom of religion in order to protect public order, morality

and health. This means that the freedom of religion is not an unlimited right. The

government can interfere in religious matters for rooting out certain social evils. So,

Statement 2 is correct.

According to Article 27, the citizens cannot be forced to pay taxes for the promotion or

maintenance of any particular religion or religious denominations. So, Statement 3 is

correct.

Q.16) With reference to fundamental rights under part III of the Constitution, which of the

following statements is/are correct?

1. All the fundamental rights are available only against arbitrary state action and not against

private individuals.

2. If rights of an individual are violated by the state, the Constitution does not provide for any

explicit mechanism for its enforcement.

3. Most of the rights are negative in character.

Select the correct answer using the code given below:

(a) 1 only

(b) 2 and 3 only

(c) 3 only

(d) 1 and 3 only

EXPLANATION

Most of the fundamental rights are available only against arbitrary state action. However,

some of them are also available against the action of private individuals. When the rights that

are available against the state’s action only or violated by private individuals, there are no

constitutional remedies but only ordinary legal remedies So, Statement 1 is not correct.

Right to Constitutional remedies under Article 32 is the explicit mechanism available for

enforcement of the fundamental rights if the rights of an individual are violated by the state.

It is the very soul of the Constitution and the very heart of it. So, Statement 2 is not

correct.

Most of the rights are negative in character that they place limitations on the authority of the

state, while others are positive in nature, conferring certain privileges on the persons. So,

Statement 3 is correct.

ADDITIONAL INFORMATION

Features of Fundamental Rights

• Some of them are available only to the citizens while others are available to all persons whether

citizens, foreigners or legal persons like corporations or companies.

• They are not absolute but qualified. The state can impose reasonable restrictions on them.

However, whether such restrictions are reasonable or not is to be decided by the courts. Thus,

they strike a balance between the rights of the individual and those of the society as a whole,

between individual liberty and social control.

• They are justiciable, allowing persons to move the courts for their enforcement, if they are

violated.

• They are defended and guaranteed by the Supreme Court. Hence, the aggrieved person can

directly go to the Supreme Court, not necessarily by way of appeal against the judgement of

the high courts.

• They are not sacrosanct or permanent. The Parliament can curtail or repeal them but only by a

constitutional amendment act and not by an ordinary act. Moreover, this can be done without

affecting the ‘basic structure’ of the Constitution. (The amenability of fundamental rights is

explained in detail in Chapter 11).

• They can be suspended during the operation of a National Emergency except the rights

guaranteed by Articles 20 and 21. Further, the six rights guaranteed by Article 19 can be

suspended only when emergency is declared on the grounds of war or external aggression (i.e.,

external emergency) and not on the ground of armed rebellion (i.e., internal emergency). (The

suspension of fundamental rights during a national Emergency is explained in detail in

Chapter 16).

• Their scope of operation is limited by Article 31A (saving of laws providing for acquisition of

estates, etc.), Article 31B (validation of certain acts and regulations included in the 9th

Schedule) and Article 31C (saving of laws giving effect to certain directive principles).

• Their application to the members of armed forces, para-military forces, police forces,

intelligence agencies and analogous services can be restricted or abrogated by the Parliament

(Article 33).

• Their application can be restricted while martial law is in force in any area. Martial law means

‘military rule’ imposed under abnormal circumstances to restore order (Article 34). It is

different from the imposition of national emergency.

• Most of them are directly enforceable (self-executory) while a few of them can be enforced on

the basis of a law made for giving effect to them. Such a law can be made only by the

Parliament and not by state legislatures so that uniformity throughout the country is

maintained (Article 35).

Q.17) Consider the following:

1. Ordinance

2. Bye- laws

3. Rules and regulations

4. Customs

Which of the above are included in the scope of ‘law’ as per article 13 of the Constitution?

(a) 1 and 2 only

(b) 1,2 and 3 only

(c) 1 and 3 only

(d) 1,2,3 and 4

EXPLANATION

The term Law’ in Article 13 has been given a wide connotation so as to include the

following:

a) Permanent laws enacted by the Parliament or the state legislatures;

b) Temporary Jaws like ordinances issued by the president or the state governors;

c) Statutory instruments in the nature of delegated legislation (executive legislation)

like order, bye-law, rule, regulation or notification; and

d) Non-legislative sources of law, that is custom or usage having the force of law

All the four are included in the scope of ‘law’ as per article 13 of the Constitution. So,

option (d) is correct.

Q.18) The term ‘Popular Sovereignty’ in the Indian context can be best understood by which of

the following statements?

(a) India is a republic with an elected head of the state

(b) The Parliament is independent to make laws for its citizens within and outside the Indian

territory.

(c) The real power of governance lies with the head of the government in India

(d) The Executive in India is collectively responsible to the legislature.

EXPLANATION

The term Popular Sovereignty can be best understood as “India is a republic with an

elected head of the state”. Popular sovereignty is a government based on the consent of

the people. The government’s source of authority is the people, and its power is not

legitimate if it disregards the will of the people. The government established by free

choice of the people is expected to serve the people, who have sovereignty, or supreme

power. So, option (a) is correct.

Q.19) Consider the following statements:

1. Rights place an obligation upon every individual to respect each other’s rights.

2. Natural rights are inalienable and inseparable from human personality.

3. Right to free trial is not a fundamental but a legal right in India.

Which of the above statements is//are correct?

(a) 1 and 2 only

(b) 2 only

(c) 2 and 3 only

(d) 1 and 3 only

EXPLANATION

Every human deserve to be respected and have the right to live with dignity. Rights place an

obligation upon every individual to respect each other’s rights which involves protection of

each other’s rights. In India it is constitutional mandate of judiciary to protect human rights of

the citizens. Supreme Court and High Courts are empowered to take action to enforce these

rights. Machinery for redress is provided under Articles 32 and 226 of the constitution. So,

Statement 1 is correct.

Natural rights are rights that are acquired by birth. The natural right includes the right to life,

liberty, dignity, and freedom. Natural rights are inalienable. They should not be taken away,

except in specific situations and according to due process. For example, the right to liberty

may be restricted if a person is found guilty of a crime by a court of law. The idea of natural

rights is inseparable from the doctrine that all human beings, regardless of extrinsic

differences in a circumstance like a nationality, class, religion, or physical condition such as

race, gender, age, etc. share an identical set of powers and freedoms. So, Statement 2 is

correct.

Right to speedy trail is a fundamental right under that Article 21 of the Indian Constitution.

But right to free trail is not a fundamental right. So, Statement 3 is not correct.

Q.20) Which of the following is/are mandated under the Right to Education Act, 2009?

1. 25 percent seats in private schools to be reserved for children belonging to economically

weaker sections.

2. A school management committee with 50 percent members being women or parents of

underprivileged children.

3. Right to completion of elementary education and a completion certificate.

Select the correct answer using the code given below:

(a) 1 and 2 only

(b) 2 and 3 only

(c) 1 and 3 only

(d) 1,2 and 3

EXPLANATION

The RTE act 2009 provided that 25 % of the seats in private schools must be reserved for the

children belonging to the Economically weaker sections and the disadvantaged categories.

So, statement 1 is correct.

The Right of Children to Free and Compulsory Education (RTE) Act 2009 insists upon

schools to constitute School Management Committees (SMCs) comprising local authority

officials, parents, guardians, and teachers. RTE also mandates the inclusion of 50 percent

women and parents of children from disadvantaged groups in SMCs. Such community

participation will be crucial to ensuring a child-friendly environment. So, Statement 2 is

correct.

The RTE act 2009 also provides the right to completion of elementary education. No child

shall be required to pass any Board examination till completion of elementary education.

Every child completing his elementary education shall be awarded a certificate in such form

and in such manner, as may be prescribed. So, Statement 3 is correct.

ADDITIONAL INFORMATION

RTE ACT 2009

• The Right of Children to Free and Compulsory Education Act or Right to Education Act (RTE),

is an Act of the Parliament of India enacted on 4 August 2009, which describes the modalities

of the importance of free and compulsory education for children between 6 and 14 in India

under Article 21a of the Indian Constitution.

• India became one of 135 countries to make education a fundamental right of every child when

the Act came into force on 1 April 2010.

• It also prohibits all unrecognized schools from practice and makes provisions for no donation

or capitation fees and no interview of the child or parent for admission.

• The Act also provides that no child shall be held back, expelled, or required to pass a board

examination until the completion of elementary education.

• The Right of Children to Free and Compulsory Education (RTE) Act, 2009 in its Schedule lays

down Pupil-Teacher Ratio (PTR) for both primary and upper primary schools. At the primary

level, the PTR should be 30:1 and at the upper primary level, it should be 35:1.

• Proof of age for admission - For the purposes of admission to elementary education, the age

of a child shall be determined on the basis of the birth certificate issued in accordance with the

provisions of the Births, Deaths and Marriages Registration Act, 1886 (6 of 1886) or on the

basis of such other document, as may be prescribed.

• No child shall be denied admission in a school for lack of age proof.

• No denial of admission.—A child shall be admitted in a school at the commencement of the

academic year or within such extended period as may be prescribed:

• Provided further that any child admitted after the extended period shall complete his studies in

such manner as may be prescribed by the appropriate Government.

• Prohibition of holding back and expulsion.—No child admitted in a school shall be held back

in any class or expelled from school till the completion of elementary education.

• Prohibition of physical punishment and mental harassment to child.— No child shall be

subjected to physical punishment or mental harassment

• The School Management Committee shall perform the following functions, namely:

a. monitor the working of the school;

b. prepare and recommend school development plan;

c. monitor the utilization of the grants received from the appropriate Government or local

authority or any other source; and

d. perform other such functions as may be prescribed

Q.21) If a person is arrested under Preventive detention law, which of the following safeguards

is/are available to him under article 22 of the Indian Constitution?

1. No detention beyond three months unless an advisory body recommends for extension.

2. Right to be produced before the nearest Magistrate within 24 hours of arrest.

3. Right to be communicated on the ground of arrest.

Select the correct answer using the code given below:

(a) 1 only

(b) 1 and 3 only

(c) 2 and 3 only

(d) 1,2 and 3

EXPLANATION

The second part of Article 22 grants protection to persons who are arrested or detained

under a preventive detention law. This protection is available to both citizens as well as

aliens and includes the following provisions

1. The detention of a person cannot exceed three months unless an advisory board reports

sufficient cause for extended detention. The board is to consist of judges of a high

court.

2. The grounds of detention should be communicated to the detenu. However, the facts

considered to be against the public interest need not be disclosed.

3. The detenu should be allowed to make a representation against the detention order.

Right to be produced before the nearest Magistrate within 24 hours of arrest is not

available to the individual detained under Preventive Detention.

So, option (b) is correct

ADDITIONAL INFORMATION

Article 22

• Article 22 grants protection to persons who are arrested or detained.

• Detention is of two types, namely, punitive and preventive. Punitive detention is to punish a

person for an offence committed by him after trial and conviction in a court.

• Preventive detention, on the other hand, means detention of a person without trial and

conviction by a court. Its purpose is not to punish a person for a past offence but to prevent

him from committing an offence in the near future.

• Thus, preventive detention is only a precautionary measure and based on suspicion.

• Article 22 has two parts—the first part deals with the cases of ordinary law and the second

part deals with the cases of preventive detention law.

• The first part of Article 22 confers the following rights on a person who is arrested or detained

under an ordinary law:

• Right to be informed of the grounds of arrest.

• Right to consult and be defended by a legal practitioner.

• Right to be produced before a magistrate within 24 hours, including the journey time.

• Right to be released after 24 hours unless the magistrate authorizes detention

• These safeguards are not available to an enemy alien or a person arrested or detained under a

preventive detention law.

Q.22) Consider the following statements:

1. The rule of law under article 14 is part of basic structure and cannot be amended by the

Parliament.

2. The rule of equality before law is an absolute right.

3. Any law made by state to implement the directive principles contained in article 39 cannot be

violative of article 14 of the Constitution.

Which of the above statements is//are correct?

(a) 1 and 2 only

(b) 3 only

(c) 2 only

(d) 1 only

EXPLANATION

In the case of Indira Nehru Gandhi v. Raj Narayan SC held that Rule of Law embodied in Article

14 of the Constitution is the “basic feature” of the Indian Constitution and hence it cannot be

destroyed even by an amendment of the Constitution under Article 368 of the Constitution. So,

Statement 1 is correct.

The rule of equality before the law is not absolute and there are exceptions to it. Example: The

President and Governor enjoys some immunities such as

1. The Persistent or Governor is not answerable to any court for the exercise and performance

of the powers and duties of his office

2. No criminal proceedings shall be initiated or continued against the President or Governor in

any court during his term of office.

So, Statement 2 is not correct.

Article 31-C is an exception to Article 14. It provides that the laws made by the state for

implementing the Directive Principles contained in clause (b) or clause (c) of Article 39 cannot be

challenged on the ground that they are violative of Article 14. The Supreme Court held that

“where Article 31-C comes in, Article 14 goes out”. So, Statement 3 is not correct.

Q.23) With reference to right to freedom of religion granted under article 28 of the

Constitution, consider the following statements:

1. Religious instruction is completely prohibited only in institutions which are wholly maintained

by the state.

2. The constitution allows for mandatory religious instructions in institutions which are

established under any endowment or trust.

Which of the above statements is//are correct?

(a) 1 only

(b) 2 only

(c) Both 1 and 2

(d) Neither 1 nor 2

EXPLANATION

Under Article 28, no religious instruction shall be provided in any educational institution

wholly maintained out of state funds. However, this shall not apply to the educational

institution administered by the state but established under any endowment or trust,

requiring imparting of religious instructions in such institution. So, Statement 1 is correct.

Religious instructions are permitted in the institution administered by the state but

established under any endowment or trust. But it is not constitutional mandatory. So,

Statement 2 is not correct.

Q.24) Consider the following statements with respect to right to Constitutional remedies

under the Indian Constitution:

1. Parliament has the power to empower any court other than the Supreme court to issue writs

for the enforcement of fundamental rights.

2. The right to move the supreme court under article 32 cannot be suspended during a national

emergency.

Which of the above statements is//are correct?

(a) 1 only

(b) 2 only

(c) Both 1 and 2

(d) Neither 1 nor 2

EXPLANATION

Parliament can empower any other court to issue directions, orders, and writs of all kinds.

However, this can be done without prejudice to the above powers conferred on the Supreme

Court. Any other court here does not include high courts because Article 226 has already

conferred these powers on the high courts. So, statement 1 is correct.

The right to move the Supreme Court shall not be suspended except as otherwise provided

for by the Constitution. Thus the Constitution provides that the President can suspend the

right to move any court for the enforcement of the fundamental rights during a national

emergency (Article 359). So, statement 2 is not correct.

Q.25) Consider the following statements:

1. The writ is issued by the court which calls upon a public authority to fulfill its public duty to

which it is bound under any enactment.

2. It can never be issued against any private individual.

3. The writ ensures that the will of the legislature is properly followed during the process of

administration.

Which of the following writs is being described above?

(a) Mandamus

(b) Prohibition

(c) Quo Warranto

(d) Certiorari

EXPLANATION

The above-mentioned writ is Mandamus.

• It means ‘we command’. It is a command issued by the court to a public official

asking him to perform the official duties that he has failed or refused to perform.

• It can also be issued against any public body, a corporation, an inferior court, a

tribunal, or government for the same purpose.

The writ of mandamus cannot be issued

1. against a private individual or body;

2. to enforce departmental instruction that does not possess statutory force;

3. when the duty is discretionary and not mandatory;

4. to enforce a contractual obligation;

5. against the president of India or the state governors; and

6. Against the chief justice of a high court acting in judicial capacity.

So, option (a) is correct.

ADDITIONAL INFORMATION

Writs

The Supreme Court shall have power to issue directions or orders or writs for the enforcement of any

of the fundamental rights. The writs issued may include habeas corpus, mandamus, prohibition,

certiorari and quo-warranto.

Habeas

Corpus

It is a Latin term which literally means ‘to have the body of. It is an order

issued by the court to a person who has detained another person, to

produce the body of the latter before it. The court then examines the cause

and legality of detention. It would set the detained person free, if the

detention is found to be illegal. Thus, this writ is a bulwark of individual

liberty against arbitrary detention.

The writ of habeas corpus can be issued against both public authorities as

well as private individuals. The writ, on the other hand, is not issued where

the (a) detention is lawful, (b) the proceeding is for contempt of a legislature

or a court, (c) detention is by a competent court, and (d) detention is

outside the jurisdiction of the court.

Prohibition Literally, it means ‘to forbid’. It is issued by a higher court to a lower court

Or tribunal to prevent the latter from exceeding its jurisdiction or usurping

a jurisdiction that it does not possess. Thus, unlike mandamus that directs

activity, the prohibition directs inactivity. The writ of prohibition can be

issued Only against judicial and quasi-judicial authorities,

It is not available against administrative authorities, legislative bodies, and

private individuals or bodies.

Certiorari In the literal sense, it means ‘to be certified’ or ‘to be informed’. It is issued

by a higher court to a lower court or tribunal either to transfer a case

pending with the latter to itself or to squash the order of the latter in a case.

It is issued on the grounds of excess of jurisdiction or lack of jurisdiction or

error of law. Thus, unlike prohibition, which is only preventive, certiorari is

both preventive as well as curative.

Previously, the writ of certiorari could be issued only against judicial and

quasi-judicial authorities and not against administrative authorities.

However, in 1991, the Supreme Court ruled that the certiorari can be

issued even against administrative authorities affecting rights of

individuals.

Like prohibition, certiorari is also not available against legislative bodies

and private individuals or bodies

Quo-

Warranto

In the literal sense, it means ‘by what authority or warrant’. It is issued by

the court to enquire into the legality of claim of a person to a public office.

Hence, it prevents illegal usurpation of public office by a person.

The writ can be issued only in case of a substantive public office of a

permanent character created by a statute or by the Constitution. It cannot

be issued in cases of ministerial office or private office.

Unlike the other four writs, this can be sought by any interested person and

not necessarily by the aggrieved person.

Q.26) With reference to Right to Property in the Indian Constitution, consider the following

statements:

1. The 42nd constitutional amendment act abolished the right to property as a fundamental right

and made it a legal right.

2. Right to property is a part of the basic structure of the Constitution.

3. The state is not mandated to provide compensation in case of acquisition of private property.

Which of the above statements is//are correct?

(a) 1 and 2 only

(b) 3 only

(c) 1 and 3 only

(d) 2 and 3 only

EXPLANATION

The 44th Amendment Act of 1978 abolished the right to property as a Fundamental Right by

repealing Article 19(1) (f) and Article 31 from Part III. Instead, the Act inserted a new Article

300A in Part XII under the heading ‘Right to Property’. It provides that no person shall be

deprived of his property except by authority of law. So, Statement 1 is not correct.

It is not a part of the basic structure of the Constitution. Thus, the right to property still

remains a legal right or a constitutional right, though no longer a fundamental right. So,

Statement 2 is not correct.

The state is not guaranteed to provide compensation in case of acquisition or requisition of

private property by the state.

However, Part III carries two provisions that provide guaranteed right to compensation in

case of acquisition and requisition of private property by the state.

a. When the State acquires the property of a minority educational institution (Article 30);

b. When the State acquires the land held by a person under his personal cultivation and the

land is within the statutory ceiling limits (Article 31 A).

The first provision was added by the 44th Amendment Act (1978), while the second provision

was added by the 17th Amendment Act (1964). So, statement 3 is correct.

Q.27) Consider the following:

1. Duty of the state government to promote instructions in mother tongue at the primary stage of

education to children belonging to linguistic minority groups.

2. Duty of the union to promote the spread of Hindi language.

Which of the above directives are mentioned under part IV of the Constitution?

(a) 1 only

(b) 2 only

(c) Both 1 and 2

(d) Neither 1 nor 2

EXPLANATION

The Constitution contains certain special directives to protect the interests of linguistic

minorities and to promote the development of Hindi language. There are:

Protection of Linguistic Minorities

In this regard, the Constitution made the following provisions:

1. It shall be the endeavour of every State and of every local authority within the State to

provide adequate facilities for instruction in the mother-tongue at the primary stage of

education to children belonging to linguistic minority groups; and the President may issue

such directions to any State as he considers necessary or proper for securing the provision

of such facilities. This provision is related to Part XVII of the Indian constitution(Official

Language)

Article 351 - It shall be the duty of the Union to promote the spread of the Hindi language, to

develop it so that it may serve as a medium of expression for all the elements of the composite

culture of India and to secure its enrichment by assimilating without interfering with its

genius, the forms, style and expressions used in Hindustani and in the other languages of

India specified in the Eighth Schedule, and by drawing, wherever necessary or desirable, for its

vocabulary, primarily on Sanskrit and secondarily on other languages. This provision is also

related to part XVII of the Indian Constitution.

Both the provisions are related to Part XVII of the Indian constitution(Official Language). So,

option (d) is correct.

ADDITIONAL INFORMATION

Official Language Act 1963

• The Official Languages Act (1963) lays down that English should be used for purposes of

communication between the Union and the non-Hindi states (that is, the states that have not

adopted Hindi as their official language).

• Further, where Hindi is used for communication between a Hindi and a non-Hindi state, such

communication in Hindi should be accompanied by an English translation.

• The Official Languages Act of 1963 lays down that Hindi translation of acts, ordinances,

orders, regulations and bye-laws published under the authority of the president is deemed to

be authoritative texts. Further, every bill introduced in the Parliament is to be accompanied by

a Hindi translation. Similarly, there is to be a Hindi translation of state acts or ordinances in

certain Cases.

• The constitutional provisions dealing with the language of the courts and legislation are as

follows:

• Until Parliament provides otherwise, the following are to be in the English language only:

o All proceedings in the Supreme Court and in every high court.

o The authoritative texts of all bills, acts, ordinances, orders, rules, regulations and bye-

laws at the Central and state levels’.

• However, the governor of a state, with the previous consent of the president, can authorise the

use of Hindi or any other official language of the state, in the proceedings in the high court of

the state, but not with respect to the judgements, decrees and orders passed by it. In other

words, the judgements, decrees and orders of the high court must continue to be in English

only (until Parliament otherwise provides).

• Similarly, a state legislature can prescribe the use of any language (other than English) with

respect to bills, acts, ordinances, orders, rules, regulations, or bye-laws, but a translation of

the same in the English language is to be published

Q.28) With reference to fundamental duties, consider the following statements:

1. Fundamental duties can be used by courts in determining the constitutional validity of a law.

2. They are not enforceable by courts but can be enforced by law.

3. Fundamental duties are applicable to both citizens and foreigners.

Which of the above statements is//are correct?

(a) 1 and 2 only

(b) 2 and 3 only

(c) 1 only

(d) 3 only

EXPLANATION

Fundamental duties help the courts in examining and determining the constitutional validity

of a law. In 1992, the Supreme Court ruled that in determining the constitutionality of any

law, if a court finds that the law in question seeks to give effect to a fundamental duty, it may

consider such law to be ‘reasonable’ in relation to Article 14 (equality before law) or Article 19

(six freedoms) and thus save such law from unconstitutionality. So, Statement 1 is

correct.

Fundamental Duties are non – Justiciable. Constitution does not provide any direct

enforcement by the courts. However, Parliament is free to enforce them by suitable legislation

and by making laws to enforce them. So, Statement 2 is correct.

Fundamental Duties are only available to the citizens and does not extend to the foreigners.

So, Statement 3 is not correct.

Q.29) Consider the following:

1. Untouchability

2. Scheduled caste

3. Religious minorities

4. Martial law

Which of the above terms have been defined in the Indian Constitution?

(a) 1 and 2 only

(b) 3 only

(c) 2 and 4 only

(d) None

EXPLANATION

The term Untouchability is not defined anywhere in the Constitution or Act.

The term Martial law is not defined in the Constitution but literally, it means 'military rule'.

Martial law is imposed under extraordinary circumstances like war, invasion, insurrection,

rebellion, riot, or any violent resistance to the law.

The term Religious minorities is not defined anywhere in the Constitution. However, the term

minorities are used in Article 29 – 30.

The term Scheduled Caste is not defined anywhere in the Constitution. Only the provisions for

establishing a community as SC are defined in the Constitution.

Thus, one of the above terms is defined in the Indian Constitution. So, option (d) is correct.

Q.30) Consider the following statements:

1. Article 1 recognises India as the official name of the country.

2. The Union of India includes all the states along with the union territories but excludes the

territories that may be acquired by the government at any time.

3. The Indian federation resembles more closely with the Canadian federation than the American

federation.

Which of the above statements is//are correct?

(a) 1 and 2 only

(b) 2 and 3 only

(c) 3 only

(d) 1,2 and 3

EXPLANATION

Article 1 recognizes India that is Bharat as the official name of the Country. There was no

unanimity in the Constituent Assembly with regard to the name of the country. Some

members suggested the traditional name (Bharat), while other advocated the modern name

(India). Hence, the Constituent Assembly had to adopt a mix of both (‘India, that is,

Bharat’)So, Statement 1 is not correct.

The Union of India includes only the states; it excludes Union territories and the territories

that may be acquired by the government at any time. The Territory of India includes all

States, Union territories and the territories that may be acquired by the government of India

at any future time. So, Statement 2 is not correct.

The Indian federation resembles more closely with the Canadian federation than the

American federation, because the Indian federation is not the result of an agreement among

the states like the American federation and the States have no rights to secede from the

federation. So, Statement 3 is correct.

Q.31) Which of the following statements are correct in Indian context?

(a) Laws made by Parliament for admission or establishment of new states are not to be

considered as amendments of the Constitution under article 368.

(b) The Indian territory can be ceded to a foreign territory by executive action and does not require

Constitutional amendment.

(c) The settlement of boundary disputes between India and another country requires amending

the Constitution under article 368.

(d) None of these are correct.

EXPLANATION

The Constitution (Article 4) itself declares that laws made for admission or establishment of new

states (under Article 2) and formation of new states and alteration of areas, boundaries, or names

of existing States (under Articles 3) are not to be considered as amendments of the Constitution

Article 368. This means that such laws can be passes by a simple majority and by the ordinary

legislative process. So, Statement 1 is correct.

The Indian territory can be ceded to a foreign territory, only by the Constitutional amendment

under Article 368. It cannot be done by Executive action. So, Statement 2 is not correct.

SC in 1968 ruled that the settlement of boundary disputes between India and another country

requires does not require Constitutional amendment under Article 368. It can be done by executive

action as it does not involve the cession of Indian Territory to a foreign country. So, Statement 3

is not correct.

Q.32) If a person wishes to acquire the Indian citizenship by naturalisation, which of the

following conditions must be fulfilled?

1. He should have denounced the citizenship of any foreign country before applying for the Indian

citizenship.

2. He should have resided in India or been in the government service throughout the period of

twelve months immediately preceding the date of application.

3. He should have an adequate knowledge of a language specified in the Eighth schedule to the

Constitution.

Select the correct answer using the code given below:

(a) 1 and 2 only

(b) 2 and 3 only

(c) 1 and 3 only

(d) 1,2 and 3

EXPLANATION

The Central Government may, On an application, grant a certificate of naturalisation to any

person (not being an illegal migrant) if he possesses the following qualifications:

a. he is not a subject or citizen of any country where citizens of India are prevented from

becoming subjects or citizens of that country by naturalisation;

b. if he is a citizen of any country, he undertakes to renounce the citizenship ( not to be

denounced) of that country in the event of his application for Indian citizenship being

accepted;

c. he has either resided in India or been in the service of a Government in India or partly

the one and partly the other, throughout the period of twelve months immediately

preceding the date of the application;

d. During the fourteen years immediately preceding the said period of twelve months, he has

either resided in India or been in the service of a Government in India, or partly the one and

partly the other, for periods amounting in the aggregate to not less than eleven years;

e. that he is of good character;

f. that he has an adequate knowledge of a language specified in the Eighth Schedule to

the Constitution and

g. that in the event of a certificate of naturalisation being granted to him, he intends to reside in

India, or to enter into or continue in, service under a Government in India or under an

international organisation of which India is a member or under a society, company or body of

persons established in India.

The Government of India may waive all or any of the above conditions for naturalisation in the

case of a person who has rendered distinguished service to science, philosophy, art, literature,

world peace or human progress. Every naturalised citizen must take an oath of allegiance to the

Constitution of India.

So, option (b) is correct.

Q.33) Which of the following benefits are entitled to an Overseas Citizen of India(OCI)

cardholder?

1. Multiple entry lifelong visas for visiting India for any purpose.

2. Exemption from registration with Foreigners regional Registration officer (FRRO) for any length

of stay in India.

3. Parity with Non resident Indians is respect of facilities in economic, financial and education

including matters related to acquisition of agricultural or plantation properties.

Select the correct answer using the code given below:

(a) 1 and 2 only

(b) 2 only

(c) 1 and 3 only

(d) 1,2 and 3

EXPLANATION

Listed here are the some of the benefits available to OCI card holders.

i. Multiple entry lifelong visa for visiting India for any purpose (However OCI

Cardholders will require Special permission to undertake research work in India for

which they may submit the application to the Indian Mission/ Post/ FRRO concerned).

ii. Exemption from registration with Foreigners Regional Registration Officer (FRRO)

or Foreigners Registration Officer (FRO) for any length of stay in India.

iii. Parity with Non-Resident Indians in respect of all facilities available to them in

economic, financial, and educational fields except in matters relating to the

acquisition of agricultural or plantation properties.

iv. Registered Overseas Citizen of India Cardholder shall be treated at par with Non-Resident

Indians in the matter of inter-country adoption of Indian children.

v. Registered Overseas Citizen of India Cardholder shall be treated at par with resident

Indian nationals in the matter of tariffs in air fares in domestic sectors in India.

vi. Registered Overseas Citizen of India Cardholders shall be charged the same entry fee as

domestic Indian visitors to visit national parks and wildlife sanctuaries in India.

vii. Parity with Non-Resident Indians (NRI) in respect of:

A. entry fees to be charged for visiting the national monuments, historical sites, and

museums in India.

B. pursuing the following professions in India, in pursuance of the provisions contained in

the relevant Acts, namely:

a. doctors, dentists, nurses and pharmacists;

b. advocates

c. architects

d. Chartered accountants.

C. to appear for the All India Pre-Medical Test or such other tests to make them eligible for

admission in pursuance of the provisions contained in the relevant Acts.

So, option (a) is correct.

ADDITIONAL INFORMATION

OCI

The Citizenship Amendment Act 2015 replaced the nomenclature of “ Overseas Citizen of India” with

that of “Overseas Citizen of India Cardholder”.

Following categories of foreign nationals are eligible for registration as Overseas Citizen of India

• Who was a citizen of India at the time of, or at any time after the commencement of the

Constitution i.e. 26.01.1950; or

• who was eligible to become a citizen of India on 26.01.1950; or who belonged to a territory that

became part of India after 15.08.1947; or

• who is a child or a grandchild or a great grandchild of such a citizen (mentioned in (1) to (3)

above) ; or

• who is a minor child of such persons mentioned above; or

• who is a minor child and whose both parents are citizens of India or one of the parents is a

citizen of India; or

• spouse of foreign origin of a citizen of India or spouse of foreign origin of an Overseas Citizen of

India Cardholder registered under section 7A of the Citizenship Act, 1955 and whose marriage

has been registered and subsisted for a continuous period of not less than two years

immediately preceding the presentation of the application.

Q.34) Consider the following pairs:

Supreme court

judgements Popular name

1. Indira Nehru Gandhi

case

- Fundamental

rights case

2. Indra Sawhney case - Mandal case

3. Kihoto Hollohan case - Defection case

4. I.R Coelho case - IX schedule case

Which of the above given pairs is/are correctly matched?

(a) 1, 2 and 3 only

(b) 3 and 4 only

(c) 2, 3 and 4 only

(d) 1 and 2 only

EXPLANATION

Q.35) With reference to the procedure for amendment of the Constitution under article 368,

consider the following statements:

1. A bill for amending the constitution can be initiated in either house of the parliament but not

in a state legislature.

2. The introduction of a constitutional amendment bill requires prior approval of the President.

3. The 24th Constitutional amendment act made the assent of the President mandatory.

1.Indira Nehru Gandhi - It is popularly known as Election Case.

1. India as a sovereign democratic republic

2. Equality of status and opportunity of an individual

3. Secularism and freedom of conscience and religion Government of laws and not of

men (i.e., Rule of Law)

4. Judicial review

5. Free and fair elections which is implied in democracy

These are some elements of basic structure declared by the SC in Indira Nehru Gandhi

case.

2. Indra Sawhney case - It is popularly known as Mandal case.

Rule of Law is the basic structure declared by the SC in Indra Sawhney Case.

3. Kihoto Hollohon Case – Popularly known as Defection case

1. Free and fair elections

2. Sovereign, democratic, republican, structure.

These are the basic structure declared by SC on Kihoto Hollon Case.

4.I.R. Coelho Case – Popularly known as IX schedule case.

1. Rule of Law

2. Separation of powers

3. Principles underlying fundamental rights

4. Judicial review

5. Principle of equality

These are basic structure declared by SC on Coelho Case.

So, option (c) is correct.

Which of the above statements is//are correct?

(a) 1 only

(b) 1 and 2 only

(c) 1 and 3 only

(d) 2 and 3 only

EXPLANATION

An Amendment of the Constitution can be initiated only by the introduction of a bill for the

purpose in either house of Parliament and not in the state Legislatures. So, Statement 1 is

correct.

The Bill can be either introduced by Minister or a private member and does not require prior

permission of the President. So, Statement 2 is not correct.

The 24th Constitutional Amendment Act made the assent of the President Obligatory. Thus

President must give his assent to the bill. He can neither withhold nor return the bill for

reconsideration of the Parliament. So, Statement 3 is correct

Q.36) For amending the provisions of the Constitution related to the federal features, which of

the following statements is/are correct?

1. The bill must be passed by a special majority of both the houses of the Parliament.

2. The bill must receive the consent of half of the state legislature by a special majority.

3. The Constitution has provided a maximum time limit of six months for the states to give their

consent to the bill.

Select the correct answer using the code given below:

(a) 1 only

(b) 1 and 2 only

(c) 2 and 3 only

(d) 1 and 3 only

EXPLANATION

Constitutional Amendment Bill must be passed in each House by a special majority that is the

majority of the total membership of the House and majority of two-third members of the

House present and voting. Each House must pass the bill separately. So, Statement 1 is

correct.

If the bill seeks to amend the federal feature of the Constitution, it must also be ratified by the

legislatures of half of the states by a simple majority (not the special majority) that is the

majority of the members of the House present and voting. So, Statement 2 is not correct.

The Constitution does not provide any time frame within which the state legislatures should

ratify or reject an amendment submitted to them. Also, Constitution does not mention

whether states can withdraw their approval after according the same. So, Statement 3 is

not correct.

Q.37) ‘Asymmetric federalism’ is an important characteristic of the Indian political system.

Which of the following provisions is/are related to it?

1. Governance of 5th and 6th schedule areas

2. Special provisions for states like Gujarat, Andhra Pradesh, etc.

3. Central government power to appoint Governors.

4. Central government power to appoint members of All India services.

Select the correct answer using the code given below:

(a) 1,2 and 3 only

(b) 1 and 2 only

(c) 3 and 4 only

(d) 1,2,3 and 4

EXPLANATION

Asymmetric federalism means federalism based on unequal powers and relationships in

political, administrative, and fiscal arrangements between the units constituting a federation.

Asymmetric federalism means different constituent states possess different powers. One or

more of the states has considerably more independence than the others, though they have the

same constitutional status. The division of powers is not symmetric.

Governance of 5th and 6th schedule areas are different from other states and more autonomy are

given to them and the special provisions are given to the states like Gujarat, Andhra Pradesh.

For Example, The President can pass an order to provide equitable opportunities and facilities

to the people belonging to different parts of Andhra Pradesh in public employment and

education.

The Appointment of the Governor to the states and Appointment of members of All India

Civil Services is a Unitary feature of the Constitution.

So, option (b) is correct.

Q.38) Which of the following is/are the necessary features of a federal government?

1. Dual government

2. Integrated judiciary

3. Division of power between national and regional government

4. Bicameral legislature

Select the correct answer using the code given below:

(a) 1,2 and 3 only

(b) 1 and 3 only

(c) 1,3 and 4 only

(d) 2 and 4 only

EXPLANATION

Listed here are the some of the necessary features of federal government

1. Dual government

2. Independent Judiciary

3. Division of power

4. Bicameral legislature

5. Written Constitution

6. Supremacy of the Constitution

7. Rigid Constitution

Integrated Judiciary is a future of a Unitary Government it is not a future of federal

government.

So, option (c) is correct.

Q.39) With respect to territorial jurisdiction of central and state governments, consider the

following statements:

1. The power of extra territorial legislation is vested in the Parliament and not the state

legislatures.

2. The President has the power to remove the application of a parliamentary law in a scheduled

area of a state.

3. The Governor can direct that a law made by parliament does not apply to the tribal area in the

state of Assam.

Which of the above statements is//are correct?

(a) 1 only

(b) 1 and 3 only

(c) 1 and 2 only

(d) 2 and 3 only

EXPLANATION

The power of extraterritorial legislation is only vested with the Parliament. Thus laws of the

Parliament are also applicable to the Indian citizens and their property in any part of the

world. The state legislatures are not vested with the power to make extraterritorial laws. So,

Statement 1 is correct.

Governor (not the President) has the power to remove the application of a parliamentary law

in a scheduled area of a state. So, Statement 2 is not correct.

The governor is empowered to direct that any particular act of Parliament or the state

legislature does not apply to a scheduled area or apply with specified modifications and

exceptions. He can also make regulations for the peace and good government of a scheduled

area after consulting the tribes advisory council. Such regulations may prohibit or restrict

the transfer of land by or among members of the scheduled tribes, regulate the allotment of

land to members of the scheduled tribes and regulate the business of money-lending in

relation to the scheduled tribes. So, Statement 3 is correct.

Q.40) Consider the following:

1. Marriage and divorce

2. Education

3. Forests

4. Census

5. Protection of wild animals and birds

Which of the above subjects are mentioned in the Concurrent list of the seventh schedule of the

Constitution?

(a) 2,3 and 5 only

(b) 1,2 and 4 only

(c) 2,4 and 5 only

(d) 1,3 and 4 only

EXPLANATION

Listed here are some of the subjects mentioned in Concurrent List.

1. Marriage and divorce; infants and minors; adoption; wills, intestacy and succession;

joint family and partition; all matters in respect of which parties in judicial

proceedings were immediately before the commencement of this Constitution subject

to their personal law

2. Prevention of cruelty to animals, Forests, Protection of wild animals and birds.

3. Education, including technical education, medical education and universities,

vocational and technical training of labour.

Census is listed in the Union list.

Q.41) Consider the following statements:

1. Reservation in promotion in favour of SCs and STs is a fundamental right under the

Constitution.

2. Parliament and not the state legislatures are empowered to make special provisions for the

advancement of any socially and educationally backward classes of citizens.

Which of the above statements is//are correct?

(a) 1 only

(b) 2 only

(c) Both 1 and 2

(d) Neither 1 nor 2

EXPLANATION

Under Article 16(4) and 16(4A) of the Constitution, Reservation in promotion in favour of SC and

ST are not fundamental rights. The Articles empower the State to make reservations in matters of

appointment and promotion in favour of the Scheduled Castes and Scheduled Tribes only “if in

the opinion of the State they are not adequately represented in the services of the State”. So,

Statement 1 is not correct.

State Legislatures are permitted to make any special provisions for the advancement of any

socially and educationally backward classes of citizens or for the SCs regarding their admission to

educational institutions whether aided or unaided by the state, except the minority educational

institutions. So, Statement 2 is not correct

Q.42) In which of the following circumstances, president assent or approval is mandatory as

per the Constitution?

1. A law made by the state legislature for acquisition of estates.

2. A state bill which seeks to take away power of a High court.

3. A state law for imposing taxation on water and electricity.

Select the correct answer using the code given below:

(a) 2 only

(b) 1 and 2 only

(c) 1 and 3 only

(d) 1,2 and 3

EXPLANATION

1. Under Article 31A the law made by the state legislature for the acquisition of estates, the

provisions of this article shall not apply, unless such law, having been reserved for the

consideration of the President, has received his assent

2. The state bill which seeks to Modify or take away the powers of HC must be reserved for

the consideration of the President and should receive his assent.

3. The Legislature of a State may by law impose, or authorize the imposition of, tax on water

and electricity, but no such law shall have any effect unless it has, after having been

reserved for the consideration of the President, received his assent; and if any such law

provides for the fixation of the rates and other incidents of such tax by means of rules or

orders to be made under the law by any authority, the law shall provide for the previous

consent of the President being obtained to the making of any such rule or order.

All the 3 bills require the President's assent or approval. So, option (d) is correct.

Q.43) Consider the following statements:

1. The doctrine of Pith and substance holds that encroachment by one legislature( centre or

states) into another in incidental matters is allowed but not in essential matters.

2. The doctrine of colourable legislation holds that indirect encroachment by one legislature into

the field assigned to another is ultra vires to the constitution.

Which of the above statements is//are correct?

(a) 1 only

(b) 2 only

(c) Both 1 and 2

(d) Neither 1 nor 2

EXPLANATION

The doctrine of Pith and substance - The doctrine states that within their respective spheres

the state and the union legislatures are made supreme, they should not encroach upon the

sphere demarcated for the other. The incidental matters are allowed in the doctrine of

Pith and substance where essential matters are not allowed.

In-State of Bombay v FN Balsara, the Bombay Prohibition Act was challenged on the ground

that it accidentally encroaches upon import and export of liquor across custom frontier – a

central subject. The court while upholding the impugned legislation declared that the Act was

in pith and substance a State subject even though it incidentally encroached upon a central

subject.So, Statement 1 is correct.

The doctrine of colourable legislation is based upon the maxim that you cannot do indirectly

what you cannot do directly and it is ultra vires to the Constitution. This is applied when the

legislature enacting the law has transgressed its power as is mentioned in the Constitution.

So, Statement 2 is correct.

Q.44) Consider the following statements:

1. Under normal circumstances, the executive power on subjects mentioned in the concurrent list

lies with the central government.

2. If there is an overlap between subjects mentioned in the concurrent list and state list, the state

list will prevail.

Which of the above statements is/are correct?

(a) 1 only

(b) 2 only

(c) Both 1 and 2

(d) Neither 1 nor 2

EXPLANATION

Under normal circumstances, the executive power on subjects mentioned in the concurrent list

lies with the state government, not with the Central Government. So, Statement 1 is not

correct

In case of conflict between the concurrent list and the state list , the concurrent list will

prevail( it is not the state list that will prevail). So, Statement 2 is not correct.

ADDITIONAL INFORMATION

Division of Powers

• The Constitution provides for a three-fold distribution of legislative subjects between the

Centre and the states, viz., List-I (the Union List), List-II (the State List) and List-III (the

Concurrent List) in the Seventh Schedule:

• The Parliament has exclusive powers to make laws with respect to any of the matters

enumerated in the Union List. This list has at present 98 subjects (originally 97° subjects) like

defence, banking, foreign affairs, currency, atomic energy, insurance, communication, inter-

state trade and commerce, census, audit and so on.

• The state legislature has “in normal circumstances” has exclusive powers to make laws with

respect to any of the matters enumerated in the State List.

• This has at present 59 subjects (originally 66 subjects) like public order, police, public health

and sanitation, agriculture, prisons, local government, fisheries, markets, theaters, gambling

and so on, Both, the Parliament and state legislature can make laws with respect to any of the

matters enumerated in the Concurrent List.

• This list has at present 52 subjects (originally 47° subjects) like criminal law and procedure,

civil procedure, marriage and divorce, population control and family planning, electricity,

labour welfare, economic and social planning, drugs, newspapers, books and printing press,

and others.

• The 42nd Amendment Act of 1976 transferred five subjects to Concurrent List from State List,

that is, (a) education, (b) forests, (c) weights and measures, (d) protection of wild animals and

birds, and (e) administration of justice; constitution and organisation of all courts except the

Supreme Court and the high courts.

• Parliament has the power to make laws with respect to any matter for any part of the territory

of India not included in a state even though that matter is one which is enumerated in the

State List. This provision has reference to the Union Territories or the Acquired Territories.

Q.45) In the context of Parliament power to make law on matters specified in state list under

article 249( National interest), which of the following statements is/are correct?

1. The resolution must be approved by the Rajya sabha members with two third majority.

2. Such resolution remains in force for one year and cannot be renewed thereafter.

3. The Parliamentary law may remain valid for indefinite time unless repealed by a competent

authority.

Select the correct answer using the code given below:

(a) 1 only

(b) 1 and 2 only

(c) 2 and 3 only

(d) 1 and 3 only

EXPLANATION

Under Article 249 If Rajya Sabha decides that it is necessary in the national interest that

Parliament should make laws with respect to any other matter in the state list then the

Parliament becomes competent to make laws on that matter. Such resolution must be

supported by 2/3rd members present and voting. So, Statement 1 is correct.

The Resolution remains in force for one year and it can be renewed any number of times but not

exceeding one year at a time. So, Statement 2 is not correct.

The law made by the parliament ceases to have effect on the expiration of six months after the

resolution has ceased to be in force. Thus law cannot remain valid for indefinite time. So,

Statement 3 is not correct.

Q.46) The delegation of executive functions of the centre to the state without the state

consent can be done by?

(a) President

(b) Governor

(c) Council of Minister

(d) Parliament

EXPLANATION

The Constitution also makes a provision for the entrustment of the executive functions of the

Centre to a state without the consent of that state. But, in this case, the delegation is by the

Parliament and not by the president. Thus, a law made by the Parliament on a subject of the

Union List can confer powers and impose duties on a state, or authorise the conferring of

powers and imposition of duties by the Centre upon a state (irrespective of the consent of the

state concerned). Notably, the same thing cannot be done by the state legislature. So, option

(d) is correct.

Q.47) With reference to grant-in- aid to the states under the Constitution, consider the

following statements:

1. Only Parliament is empowered by the Constitution to make statutory grants to the states

which are in need of financial assistance.

2. Statutory grants are given to the states on the recommendations of the Finance commission.

3. Discretionary grants given by the centre to the states form the larger part of the central grants

to the states.

Which of the above statements is/are correct?

(a) 1 and 2 only

(b) 2 and 3 only

(c) 1 and 3 only

(d) 1,2 and 3

EXPLANATION

Article 275 empowers the Parliament to make grants to the states which are in need of

financial assistance and not to every state. Also, different sums may be fixed for different

states. These sums are charged on the Consolidated Fund of India every year. So, Statement

1 is correct.

The statutory grants under Article 275 (both general and specific) are given to the states on

the recommendation of the Finance Commission. So, Statement 2 is correct.

Discretionary Grants Article 282 empowers both the Centre and the states to make any grants

for any public purpose, even if it is not within their respective legislative competence. Under

this provision, the Centre makes grants to the states.

“These grants are also known as discretionary grants, the reason being that the Centre is

under no obligation to give these grants and the matter lies within its discretion. These grants

have a two-fold purpose: to help the state financially to fulfill plan targets; and to give some

leverage to the Centre to influence and coordinate state action to effectuate the national plan.

Notably, Discretionary grants form the larger part of grants from the Central Government to

the states when compared to the Statutory grants. So, Statement 3 is correct.

Q.48) Which of the following committee/resolutions is/are related to the reforms in the

centre- state relations in Indian context?

1. Rajamanar committee

2. Anandpur Sahib resolution

3. Balwant Rai Mehta committee

4. G.V.K Rao committee

Select the correct answer using the code given below:

(a) 1,2 and 3 only

(b) 1 and 2 only

(c) 1 and 3 only

(d) 3 and 4 only

EXPLANATION

The committee related to the Center-state relations are

1. Rajamanar committee - In 1969, the Tamil Nadu Government (DMK) appointed a three-

member committee under the chairmanship of Dr. P.V. Rajamannar to examine the entire

question of Centre-state relations and to suggest amendments to the Constitution to

secure utmost autonomy to the states. The committee submitted its report to the Tamil

Nadu Government in 1971.

2. Anandpur Sahib resolution - In 1973, the Alkali Dal adopted a resolution containing

both political and religious demands in a meeting held at Anandpur Sahib in Punjab. The

resolution, generally known as Anandpur Sahib Resolution, demanded that the Centre’s

jurisdiction should be restricted only to defence, foreign affairs, communications, and

currency and the entire residuary powers should be vested in the states. It stated that

the Constitution should be made federal in the real sense and should ensure equal

authority and representation to all the states at the Centre.

Balwant Rai Mehta committee and G.V.K Rao committee are the commissions formed for the

purposes of Panchayat raj institutions.

So, option (b) is correct.

ADDITIONAL INFORMATION

Other Committees related to centre - state relations

West Bengal Memorandum - In 1977, the West Bengal Government (led by the Communists)

published a memorandum on Centre-state relations and sent to the Central government. The

memorandum inter alia suggested the following: (i) The word ‘union’ in the Constitution should be

replaced by the word ‘federal’; (ii) The jurisdiction of the Centre should be confined to defence, foreign

affairs, currency, communications and economic co-ordination; (iii) All other subjects including the

residuary should be vested in the states; (iv) Articles 356 and 357 (President's Rule) and 360 (financial

emergency) should be repealed; (v) State’s consent should be made obligatory for the formation of new

states or reorganization of existing States (vi) Of the total revenue raised by the Centre from all

sources, 75 percent should be allocated to the states; (vii) Rajya Sabha should have equal powers with

that of the Lok Sabha; and (viii) There should be only Central and state services and all India services

should be abolished.

The Central government did not accept the demands made in the memorandum.

Sarkaria Commission - In 1983, the Central government appointed a three-member Commission on

Centre state relations under the chairmanship of R.S. Sarkaria, a retired judge of the Supreme Court.”

The commission was asked to examine and review the working of existing arrangements between the

Centre and states in all spheres and recommend appropriate changes and measures. It was initially

given one year to complete its work, but its term was extended four times. It submitted it's report in

1988.

Punchhi Commission - The Second commission on Centre-State relations was set-up by the

Government of India in April 2007 under the Chairmanship of Madan Mohan Punchhi, former Chief

Justice of India.” It was required to look into the issues of Centre-State relations keeping in view the

sea-changes that have taken place in the polity and economy of India since the Sarkaria Commission

had last looked at the issue of Centre-State relations over two decades ago.

Q.49) Consider the following statements with regard to ‘Anticipatory bail’ in the Indian legal

context:

1. Anticipatory bail is a direction to release a person on bail, issued even before the person is

arrested.

2. A person can file an anticipatory bail plea only in the high court or the Supreme court of India.

Which of the above statements is/are correct?

(a) 1 only

(b) 2 only

(c) Both 1 and 2

(d) Neither 1 nor 2

EXPLANATION

Anticipatory bail can be obtained by a person who anticipates arrest. Hence, anticipatory bail

is a direction to release a person on bail, even before the person is arrested. Anticipatory bail

is applied for under Section 438 of the Criminal Procedure Code. According to Code of

Criminal Procedure, 1973, “When any person has reason to believe that he may be arrested

on an accusation of having committed a non-bailable offence, he may apply to the High Court

or the Court of Session for a direction under this section and, if the court thinks fit, direct

that in the event of such arrest, he shall be released on bail.” So, Statement 1 is correct.

According to the Code of Criminal Procedure, 1973, the provision empowers only the Sessions

Court and High Court to grant anticipatory bail. So, Statement 2 is not correct.

ADDITIONAL INFORMATION

Anticipatory bail

• ‘Bail’ is a document procuring “the release of a person from legal custody, by undertaking that

he shall appear at the time and place designated and submit himself to the jurisdiction and

judgement of the court.”

• As opposed to ordinary bail, which is granted to a person who is under arrest, in anticipatory

bail, a person is directed to be released on bail even before arrest made.

• Anticipatory bail became part of the new CrPC in 1973 (when the latter replaced the older Code

of 1898), after the 41st Law Commission Report of 1969 recommended the inclusion of the

provision.

• While granting anticipatory bail, the Sessions Court or High Court can impose the following

conditions

1. A condition that the person shall make himself available for interrogation by a police

officer as and when required;

2. A condition that the person shall not, directly or indirectly, make any inducement,

threat or promise to any person acquainted with the facts of the case so as to dissuade

him from disclosing such facts to the Court or to any police officer.

3. A condition that the person shall not leave India without the previous permission of the

Court.

4. In addition, such other condition as may be imposed under sub section (3) of section

437, as if the bail were granted under that section.

• Recently Supreme Court ruled that no time restriction should ordinarily be fixed for

anticipatory bail and that it can continue even until the end of the trial.

Q.50) With reference to provisions related to proclamation of National emergency , consider

the following statements:

1. The President can impose an emergency only after a written consent of Union cabinet headed by

the Prime Minister

2. The decision to impose national emergency is immune to judicial review.

3. It should be approved by both the houses of the Parliament within one month of the

proclamation with a special majority.

Which of the above statements is/are correct?

(a) 1 and 2 only

(b) 1 and 3 only

(c) 3 only

(d) 1,2 and 3

EXPLANATION

The President can proclaim a National emergency only after receiving a written

recommendation from the Union Cabinet headed by the Prime Minister. This means that an

emergency can be declared only on the concurrence of the Cabinet and not merely on the

advice of the Prime Minister. So, Statement 1 is correct

The SC Court in Minerva mills Case held that the proclamation of National emergency can be

challenged in a court on the ground of malaise or that the declaration was based on wholly

extraneous and irrelevant facts or is absurd or perverse. Thus The decision to impose a

national emergency is not immune from judicial review. So, Statement 2 is not correct.

The proclamation of Emergency ‘must be approved by both the Houses of Parliament within

one month from the date of its issue. Originally, the period allowed for approval by the

Parliament was two months but was reduced by the 44th Amendment Act of 1978. However, if

the proclamation of emergency is issued at a time when the Lok Sabha has been dissolved or

the dissolution of the Lok Sabha takes place during the period of one month without

approving the proclamation, then the proclamation survives until 30 days from the first sitting

of the Lok Sabha after its reconstitution provided the Rajya Sabha has in the meantime

approved it. Every resolution approving the proclamation of emergency or its continuance

must be passed by either House of Parliament by a special majority, that is, (a) a majority

of the total membership of that house, and (b) a majority of not less than two-thirds of the

members of that house present and voting. This special majority provision was introduced by

the 44th Amendment Act of 1978. Previously, such resolution could be passed by a simple

majority of the Parliament. So, Statement 3 is correct.

Q.51) Which of the following statements is incorrect?

(a) A proclamation of national emergency leads to dissolution of the state legislative

assembly.

(b) Parliament is empowered to legislate on state list item when national emergency is in operation,

(c) Fundamental right under article 19 automatically gets suspended when emergency is declared

on grounds of external aggression.

(d) All the statements are correct.

EXPLANATION

The state governments are brought under the complete control of the Centre, though the

state legislatures are not suspended. So, Statement (a) is not correct.

The parliament becomes empowered to make laws on any subject mentioned in the state

list, when emergency is in operation. The president can issue ordinances on State subjects

also, if the parliament is not in session. The laws made on state subjects by the parliament

become inoperative six months after the emergency has ceased to be in operation.

Suspension of Fundamental Rights under Article 19 -According to Article 358, when a

proclamation of national emergency is made, the six Fundamental Rights under Article

19 can be suspended only when the National Emergency is declared on the ground of

war or external aggression and not on the ground of armed rebellion. This feature was

added in the 44th Constitutional Amendment Act.

Q.52) Consider the following:

1. The elected members of both the houses of Parliament.

2. The nominated members of Lok sabha

3. The elected members of State legislative assembly

4. The elected members of the legislative assemblies of Delhi and Puducherry.

Which of the above form the electoral college for the election of the President in India?

(a) 1,2 and 3 only

(b) 1 and 3 only

(c) 1,3 and 4 only

(d) 1 and 4 only

EXPLANATION

• The President’s election is held in accordance with the system of proportional representation by a means of a single transferable vote.

• Voting is done by secret ballot.

• The electoral college of President Consist of

1. The elected members of both the Houses of Parliament.

2. The elected members of the State legislative assembly

3. The elected members of the legislative assemblies of Delhi and Puducherry.

• The nominated members of both Houses of Parliament, the nominated members of the

state legislative assemblies, the members (both elected and nominated) of the state

legislative councils (in case of the bicameral legislature) and the nominated members of

the Legislative Assemblies of Delhi and Puducherry do not participate in the election of

the President.

So, option (c) is correct.

Q.53) In the Indian context, the President can resign from the office by submitting his

resignation to?

(a) Prime Minister

(b) Chief Justice of India

(c) Vice President

(d) Speaker of Lok Sabha

EXPLANATION

President holds office for a term of five years from the date on which he enters his office. He

can resign from his office at any time by addressing the resignation letter to the Vice –

President. So, option (c) is correct.

Q.54) Which of the following veto powers is/are enjoyed by the Indian President?

1. Absolute veto

2. Qualified veto

3. Suspensive veto

4. Pocket veto

Select the correct answer using the code given below:

(a) 1,2 and 3 only

(b) 1 and 4 only

(c) 1,3 and 4 only

(d) 2 and 3 only

EXPLANATION

The veto power enjoyed by the executive in modem states can be classified into the following

four types:

1, Absolute veto, that is, withholding of assent to the bill passed by the legislature,

2. Qualified veto, which can be overridden by the legislature with a higher majority,

3. Suspensive veto, which can be overridden by the legislature with an ordinary majority.

4. Pocket veto, that is, taking no action on the bill passed by the legislature.

Of the above four, the President of India is vested with three absolute veto, suspensive veto

and pocket veto. There is no qualified veto in the case of the Indian President; it is possessed

by the American President. So, option (c) is correct.

ADDITIONAL INFORMATION

VETO POWERS

Absolute Veto

• It refers to the power of the President to withhold his assent to a bill passed by the Parliament.

The bill then ends and does not become an act. Usually, this veto is exercised in the following

two cases:

o With respect to private members’ bills (ie, bills introduced by any member of Parliament

who is not a minister); and

o With respect to the government bills when the cabinet resigns (after the passage of the

bills but before the assent by the President) and the new cabinet advises the President

not to give his assent to such bills.

Suspensive Veto

• The President exercises this veto when he returns a bill for reconsideration of the Parliament.

However, if the bill is passed again by the Parliament with or without amendments and again

presented to the President, the President must give his assent to the bill.

• This means that the presidential veto is overridden by a re-passage of the bill by the same

ordinary majority (and not a higher majority as required in the USA).

• As mentioned earlier, the President does not possess this veto in the case of money bills. The

President can either give his assent to a money bill or withhold his assent to a money bill but

cannot return it for the reconsideration of the Parliament.

• Normally, the President gives his assent to the money bill as it is introduced in the Parliament

with his previous permission.

Pocket Veto

• In this case, the President neither ratifies nor rejects nor returns the bill, but simply keeps the

bill pending for an indefinite period.

• This power of the President not to take any action (either positive or negative) on the bill is

known as the pocket veto.

• The President can exercise this veto power as the Constitution does not prescribe any time

limit within which he has to take the decision with respect to a bill presented to him for his

assent.

• In the USA, on the other hand, the President has to return the bill for reconsideration within

10 days.

• Hence, it is remarked that the pocket of the Indian President is bigger than that of the

American President.

Q.55) Which of the following statements is correct with respect to the removal process of the

Vice -President in India?

(a) A resolution of the Rajya Sabha passed by a special majority and by the Lok sabha with an

absolute majority.

(b) A resolution of the Rajya Sabha passed by an absolute majority and agreed to by the Lok

sabha with a simple majority.

(c) A resolution of the Rajya Sabha passed by a simple majority and agreed to by the Lok Sabha

with a simple majority.

(d) A resolution of the Rajya Sabha passed by an effective majority and by the Lok sabha by an

effective majority.

EXPLANATION

• The Vice-President holds office for a term of five years from the date on which he enters his

office.

• He can resign from his office at any time by addressing the resignation letter to the

President. He can also be removed from the office before the completion of his term. A

formal impeachment is required for his removal.

• He can be removed by a resolution passed by a majority of all the then members of

the Rajya Sabha and agreed to by the Lok Sabha.

• This means that this resolution should be passed in the Rajya Sabha by an absolute

majority and in the Lok Sabha by a simple majority.

• Further, this resolution can be introduced only in the Rajya Sabha and not in the Lok

Sabha. But, no such resolution can be moved unless at least 14 days advance notice has

been given. So, option (b) is correct.

Q.56) Consider the following statements:

1. In India, the Prime Minister may be a member of either house of the Parliament, while he must

be a member of the lower house in Britain.

2. The President dissolves the lok sabha on the recommendations of the Prime Minister.

3. The resignation of an incumbent Prime Minister leads to automatic dissolution of lok sabha.

Which of the above statements is/are correct?

(a) 1 and 2 only

(b) 1 only

(c) 2 and 3 only

(d) 1,2 and 3

EXPLANATION

Constitutionally, the Prime Minister may be a member of any of the two Houses of parliament.

In Britain, the Prime Minister should be a member of the Lower House (House of Commons).

So, Statement 1 is correct

President can dissolve the Lok Sabha on the advice of the Council of Ministers headed by the

Prime Minister. As PM is the head of the Council of Ministers and represents the Council so

recommendations can be made by the Prime Minister to President to dissolve the Lok Sabha.

So, Statement 2 is correct.

Prime Minister stands at the head of the council of ministers, the other ministers cannot

function when the Prime Minister resigns or dies. The resignation or death of an incumbent

Prime Minister automatically dissolves the council of ministers and thereby generates a

vacuum. The resignation of an incumbent Prime Minister does not leads to automatic

dissolution of lok sabha. So, Statement 3 is not correct.

Q.57) Which of the following offices is most instrumental in upholding the principle of

Collective responsibility enshrined in article 75 of the Constitution?

(a) President

(b) Speaker of Lok sabha

(c) Prime Minister

(d) Chief justice of Supreme court

EXPLANATION

The Council ministers are collectively responsible to the Lok Sabha and Prime minister is the

head of the council of ministers. Thus Prime Minister is most instrumental in upholding the

principle of Collective responsibility enshrined in article 75 of the Constitution. So option (c)

is correct.

Q.58) Consider the following statements with respect to ‘Parliamentary secretary’ :

1. It is a Constitutional office and is appointed by the Prime Minister to assist the cabinet

ministers in the discharge of their parliamentary duties.

2. No Parliamentary secretary has been appointed in India since independence.

Which of the above statements is/are correct?

(a) 1 only

(b) 2 only

(c) Both 1 and 2

(d) Neither 1 nor 2

EXPLANATION

Parliamentary Secretaries have in the past been appointed by the Prime Minister from time to

time. They were administered an oath of secrecy only and no office. Parliamentary Secretaries

work in an honorary capacity. They have not been given any administrative work but have been

appointed to assist the Minister in his Parliamentary and political work. The institution of the

office of Parliamentary Secretary has no statutory origin nor does it derive authority

from the Constitution of India. The Office of the Parliamentary Secretary in India was first

created in 1951. So, Statement 1 is not correct.

Parliamentary Secretaries are appointed since Independence. Manipur, Himachal Pradesh,

Mizoram, Assam, Rajasthan, Punjab, Goa are some of the states where MLAs have been

appointed as Parliament Secretaries by the Government. So, Statement 2 is not correct.

ADDITIONAL INFORMATION

Parliamentary secretary

• According to the original concept in 1951, Parliamentary Secretaries were appointed purely to

assist the Minister in parliamentary work, analogous to the position of Parliamentary Private

Secretaries in the United Kingdom.

• Unlike the Ministers, the oath is administered to the Parliamentary Secretaries by

the Prime Minister and not by the President.

• The appointment of a Parliamentary Secretary is entirely the discretion of the Prime Minister

who will decide when and who should be appointed as Parliamentary Secretary.

• The Salary, allowances and other pre-requisites will be decided by the Government

from time to time keeping in view various factors including the exigencies of duties that may

require to be performed by the Parliamentary Secretary.

• The position regarding the status, powers, and functions of the Parliamentary Secretary is well

set by the then Prime Minister.

Q.59) Which of the following statements is/are correct?

1. The dissolution of lok sabha leads to automatic dissolution of the Council of ministers.

2. The advice tendered by the council of ministers to the President can not be inquired into any

court.

3. The 91st amendment act capped the total number of ministers in the council of ministers to 15

percent of the total strength of the Lok sabha.

Select the correct answer using the code given below:

(a) 1 only

(b) 2 and 3 only

(c) 1 and 3 only

(d) 2 only

EXPLANATION

Even after the dissolution of Lok Sabha the COMs will continue till the next election is done

and a new government is sworn. Thus the Council of Ministers after the dissolution of Lok

Sabha act as a care taken government, thus council of minister will just attend day to day

activities of the Government but will not take any decisions on important policy matters. So,

Statement 1 is not correct.

Article 84 deals with the Council of Ministers to aid and advise the President. There shall be a

Council of Ministers with the Prime Minister at the head to aid and advise the President who

shall, in the exercise of his functions, act in accordance with such advice. However, the

President may require the Council of Ministers to reconsider such advice and the President

shall act in accordance with the advice tendered after such reconsideration. The advice

tendered by Ministers to the President shall not be inquired into in any court. So,

Statement 2 is correct.

The total number of ministers, including the Prime Minister, in the Council of Ministers, shall

not exceed 15% of the total strength of the Lok Sabha. This provision was added by the 91st

Amendment Act of 2003. So, Statement 3 is correct.

Q.60) With reference to ‘Cabinet’ in the Indian Parliamentary system, consider the following

statements:

1. The Constitution defines the term cabinet without specifying its powers and functions.

2. The functions of the cabinet are decided by the Council of Ministers.

3. Cabinet exercises the power to enforce the collective responsibility of the council of ministers to

the lower house of the Parliament.

Which of the above statements is/are not correct?

(a) 1 only

(b) 2 only

(c) 3 only

(d) None

EXPLANATION

Article 352 defines the Cabinet as ‘the council consisting of the Prime Minister and other

ministers of Cabinet rank appointed under Article 75 , it was inserted by 44th Constitutional

Amendment Act and the powers and functions are not described in the Constitution. So,

Statement 1 is correct.

Cabinet is highest decision making body. The functions of the council of ministers are decided by

the cabinet. COMs do not decide the function of Cabinet. So, Statement 2 is not correct.

The Cabinet enforces the collective responsibility of the council of ministers to the lower house of

the Parliament. So, Statement 3 is correct.

Q.61) With reference to Zonal councils, consider the following statements:

1. Zonal councils are Constitutional bodies for the purpose of inter-state coordination.

2. The Union home minister acts as the chairman of each zonal council.

3. North East council was created by a separate act of Parliament.

Which of the above statements is/are correct?

(a) 1 only

(b) 2 and 3 only

(c) 1 and 3 only

(d) 1,2 and 3

EXPLANATION

The Zonal councils are not a Constitutional Body rather it is a statutory body established by

an Act of the Parliament called the States Reorganisation Act of 1956 for inter-state

coordination. . The Act divided the country into five zones such as Northern, Central, Eastern,

Western, and Southern. So, Statement 1 is not correct.

The Union Home Minister acts as a common chairman of the 5 zonal councils. Each Chief

Minister act as vice-chairman of the Council by rotation, holding office for one year at a time.

So, Statement 2 is correct.

North-Eastern Council was created by a separate Act of Parliament the North-Eastern Council

Act of 1971.° Its members include Assam, Manipur, Mizoram, Arunachal Pradesh, Nagaland,

Meghalaya, Tripura and Sikkim. So, Statement 3 is correct.

ADDITIONAL INFORMATION

• Each zonal council consists of the following members:

a. home minister of Central government.

b. chief ministers of all the States in the zone.

c. Two other ministers from each state in the zone.

d. Administrator of each union territory in the zone.

• Besides, the following persons can be associated with the zonal council as advisors (i.e,

without the right to vote in the meetings):

i. person nominated by the Planning Commission; (ii) chief secretary of the government of

each state in the zone; and (iii) development commissioner of each state in the zone.

• The zonal councils aim at promoting cooperation and coordination between states, union

territories, and the Centre. They discuss and make recommendations regarding matters like

economic and social planning, linguistic minorities, border disputes, interstate transport, and

so on. They are only deliberative and advisory bodies.

Q.62) The President of India is not a member of either house of the Parliament but is

considered an integral part of the Parliament. Which of the following is the most

appropriate reason for it?

(a) The President administers oaths to the Prime Minister and the council of ministers.

(b) The President has the power to summon or prorogue both the houses of the Parliament.

(c) Any bill passed by both the houses of Parliament cannot become a law without his

assent.

(d) He addresses the Parliament at the commencement of the first session after each general

election and first session of each year.

EXPLANATION

The President of India is not a member of either house of the Parliament but is considered as a

part of both executive and the legislature because all the bills passed by the Parliament become

law only when it signed by the President. So, option (c) is correct.

Q.63) With reference to composition of Lok Sabha and Rajya Sabha, consider the following

statements:

1. While the representatives of Union territories in Rajya sabha are indirectly elected, they are

directly elected in Lok sabha.

2. The nominated members in both the houses must have special knowledge or practical

experience in art, science, literature and social service.

3. The third schedule to the Constitution deals with the allocation of seats in the Rajya sabha to

the states and union territories.

Which of the above statements is/are correct?

(a) 1 and 2 only

(b) 1 only

(c) 2 and 3 only

(d) 1 and 3 only

EXPLANATION

The Representatives of Union Territory in Rajya Sabha are elected indirectly by the members of

the electoral college specially constituted for this purpose. In Lok Sabha representatives are

directly elected by the people. So, Statement 1 is correct

The Rajya Sabha has 12 nominated members who have special knowledge in art, science,

literature, and social service. While in Lok Sabha two members are nominated from the Anglo

Indian Community they no need to know about art, science, literature, and social service.

Further the nomination of Anglo Indian Community provision has ended in 2020. So statement

2 is not correct.

The fourth Schedule of the Constitution deals with the allocation of seats in the Rajya Sabha

to the states and union territories. So, Statement 3 is not correct.

Q.64) Consider the following statements:

1. The election of members to the lok sabha is based on territorial representation while that of

Rajya sabha is based on proportional representation.

2. While territorial representation does not represent the whole electorate, the proportional

representation does not allow bye- election

Which of the above statements is/are correct?

(a) 1 only

(b) 2 only

(c) Both 1 and 2

(d) Neither 1 nor 2

EXPLANATION

The election to the members of Lok Sabha is based on the system of territorial

representation. Under territorial representation, every member of the legislature represents a

geographical area known as a constituency. Representation of States The representatives of

states in the Rajya Sabha are elected by the elected members of state legislative assemblies.

The election is held in accordance with the system of proportional representation by means of

the single transferable vote. The seats are allotted to the states in the Rajya Sabha based on

population. So, Statement 1 is correct.

In the system of Territorial representation candidate who secures the majority of votes is

declared elected. This simple majority system of representation doesn’t represent the whole

electorate. It does not secure due representation to minorities.

The system of proportional representation has the following demerits

• It is highly expensive.

• It does not give any scope for organizing by-elections.

• It eliminates intimate contacts between voters and representatives.

• It promotes minority thinking and group interests.

• It increases the significance of the party system and decreases that of a voter.

So, Statement 2 is correct.

Q.65) Which of the following disqualifications for a member of Parliament is mentioned in the

Constitution?

1. Office of profit

2. Undischarged insolvent.

3. Conviction for any offence for two or more years.

4. Director in a company where the government has at least 25 percent share.

Select the correct answer using the code given below:

(a) 1 and 2 only

(b) 1,3 and 4 only

(c) 2 and 3 only

(d) 1 and 4 only

EXPLANATION

Under the Constitution, a person shall be disqualified for being elected as a member of

Parliament:

• if he holds any office of profit under the Union or state government (except that of a

minister or any other office exempted by Parliament).

• if he is of unsound mind and stands so declared by a court.

• if he is an undischarged insolvent.

• if he is not a citizen of India or has voluntarily acquired the citizenship of a foreign state or

is under any acknowledgment of allegiance to a foreign state; and if he is so disqualified

under any law made by Parliament.

The other two provisions

i. A conviction for any offense for two or more years.

ii. Director in a company where the government has at least 25 percent share.

They are mentioned in the Representation of People’s Act 1951.

So, option (a) is correct

Q.66) Consider the following statements with respect to Office of Speaker under the Indian

Constitution:

1. The Speaker is removed by a majority of not less than two third of the members of the Lok

sabha.

2. When a resolution for the removal of the speaker is under consideration, he cannot take part

in the proceedings of the house.

3. The Speaker can also offer a voluntary resignation to the Deputy Speaker of the Lok sabha.

Which of the above statements is/are correct?

(a) 1 and 3 only

(b) 2 and 3 only

(c) 3 only

(d) 1 and 2 only

EXPLANATION

Speaker can be removed only by a resolution passed by the Lok Sabha by a special majority a

majority of all the then members of the House and not by an ordinary majority. This motion of

removal can be considered and discussed only when it has the support of at least 50 members.

So, Statement 1 is correct.

When a resolution for the removal of the Speaker is under consideration of the House, he cannot

preside at the sitting of the House, though he may be present. However, he can speak and take

part in the proceedings of the House at such a time and vote in the first instance, though not

in the case of an equality of votes. So, Statement 2 is not correct.

Speaker can give voluntary resignation to the Deputy Speaker of the Lok Sabha. So, Statement

3 is correct.

Q.67) Consider the following statements:

1. Leader of the Lok sabha is a constitutional office while leader of opposition is a statutory office.

2. The leader of the house in Rajya sabha is a minister and a member of the house who is

nominated by the Prime Minister.

3. The office of Whip is neither mentioned in the Constitution nor the rules of the house.

Which of the above statements is/are correct?

(a) 1 and 2 only

(b) 2 and 3 only

(c) 1 and 3 only

(d) 1,2 and 3

EXPLANATION

Both the Leader of the Lok and the Leader of Opposition are not mentioned in the

Constitution of India, they are mentioned in the Rules of the House and Parliamentary

Statute. The leader of Opposition in the Lok Sabha and the Rajya Sabha were accorded

statutory recognition in 1977. They are also entitled to the salary, allowances, and other

facilities equivalent to that of a cabinet minister. So, Statement 1 is not correct.

There is also a Leader of House in the Rajya Sabha. He is a minister and a member of the Rajya

Sabha and is nominated by the Prime Minister to function as such. So, Statement 2 is

correct.

The office of the whip is mentioned neither in the Constitution of India nor in the Rules of the

House nor a Parliamentary Statute. It is based on the conventions of the parliamentary

governments. So, Statement 3 is correct.

Q.68) With reference to effect of Lok sabha dissolution on business of the house, consider the

following statements:

1. All pending assurances that are to be examined by the Committee on government assurances

lapse on the dissolution of lok sabha.

2. A bill does not lapse if it has been passed by both the house of the Parliament and President

assent is awaited.

Which of the above statements is/are correct?

(a) 1 only

(b) 2 only

(c) Both 1 and 2

(d) Neither 1 nor 2

EXPLANATION

All pending assurances that are to be examined by the Committee on government

assurances do not lapse on the dissolution of Lok Sabha. So, Statement 1 is not correct.

A bill passed by both the houses but pending assent of the President does not lapse. So,

Statement 2 is correct.

ADDITIONAL INFORMATION

Bills that lapses on the dissolution of Lok Sabha

• A bill pending in the Lok Sabha lapses (whether originating in the Lok Sabha or transmitted to

it by the Rajya Sabha), -

• A bill passed by the Lok Sabha but pending in the Rajya Sabha lapses.

Bills that don’t lapse on the dissolution of Lok Sabha

• A bill not passed by the two Houses due to disagreement and if the president has notified the

holding of a joint sitting before the dissolution of Lok Sabha, does not lapse.

• A bill pending in the Rajya Sabha but not passed by the Lok Sabha does not lapse.

• A bill passed by both Houses but pending assent of the president does not lapse. . A bill passed

by both Houses but returned by the president for reconsideration of Houses does not lapse,

Q.69) Consider the following statements:

1. This motion can be introduced only in the lok sabha to draw attention of the house to a

definite matter of urgent importance.

2. It needs the support of 50 members to be admitted.

3. If this motion is not passed, it can lead to defeat of the government in the house.

Which of the following motions is being described above?

(a) Adjournment motion

(b) Calling attention motion

(c) Censure motion

(d) Privilege motion

EXPLANATION

Adjournment motion is introduced only in the Lok Sabha to draw the attention of the House to a

definite matter of urgent public importance.

• It involves an element of censure against the government, therefore Rajya Sabha is not

permitted to make use of this device

• It is regarded as an extraordinary device as it interrupts the normal business of the

House. It needs the support of 50 members to be admitted.

• The discussion on this motion should last for not less than two hours and thirty

minutes.

However, the right to move a motion for an adjournment of the business of the House is subject

to the following restrictions. i.e. It should:

• Raise a matter which is definite, factual, urgent, and of public importance.

• Not cover more than one matter.

• Be restricted to a specific matter of recent occurrence.

• Not raise a question of privilege.

• Not revive discussion on a matter that has been discussed in the same session.

• Not deal with any matter that is under adjudication of court.

• Not raise any question that can be raised on a distinct motion.

So, option (a) is correct.

Q.70) Consider the following statements:

1. A starred question requires a written answer and any supplementary question cannot follow it.

2. An unstarred question is printed in white while a starred question is printed in green

3. A short notice question is answered orally and it requires giving a notice of less than ten days.

Which of the above statements is/are correct?

(a) 1 and 2 only

(b) 1 and 3 only

(c) 2 and 3 only

(d) 1,2 and 3

EXPLANATION

A Starred Question is one to which a member desires an oral answer from the Minister in the

House and is required to be distinguished by him/her with an asterisk. Answer to such a

question may be followed by supplementary questions by members. So, Statement 1 is not

correct.

An Unstarred question that requires a written answer is printed in white while the starred

question is printed in green. So, Statement 2 is correct.

Short Notice Questions- A member may give notice of question on a matter of public

importance and of urgent character for an oral answer at a notice less than 10 days prescribed

as the minimum period of notice for asking a question in an ordinary course. So, Statement

3 is correct.

ADDITIONAL INFORMATION

Question Hour

• Generally, the first hour of a sitting of Lok Sabha is devoted to the Questions and this hour is

called the Question Hour.

• Asking questions is an inherent and unfettered parliamentary right of members. It is during

the Question Hour that the members can ask questions on every aspect of administration and

Governmental activity.

• Government policies in national as well as international spheres come into sharp focus as the

members try to elicit pertinent information during the Question Hour.

• The Government is, as it were, put on its trial during the Question Hour and every Minister

whose turn it is to answer questions has to stand up and answer for his or his administration's

acts of omission and commission.

• Through the Question Hour, the Government can quickly feel the pulse of the nation and adapt

its policies and actions accordingly.

• It is through questions in the Parliament that the Government remains in touch with the

people in as much as members are enabled thereby to ventilate the grievances of the public in

matters concerning the administration.

• Questions enable Ministries to gauge the popular reaction to their policy and administration.

Questions bring to the notice of the Minister's many loopholes which otherwise would have

gone unnoticed.

• Sometimes questions may lead to the appointment of a Commission, a Court of Enquiry, or

even Legislation when matters raised by Members are grave enough to agitate the public mind

and are of wide public importance.

Q.71) With reference to ‘Resolutions’ as a device of Parliamentary proceedings, which of the

following statements is/are correct?

1. All resolutions are a type of substantive motion and need to be voted upon by the members of

the house.

2. A resolution once moved by any member cannot be withdrawn except by the leave of the

house.

3. A statutory resolution can be moved only by a minister.

Select the correct answer using the code given below:

(a) 1 and 2 only

(b) 2 only

(c) 1 and 3 only

(d) 3 only

EXPLANATION

All resolutions come into the category of Substantive motion, every resolution is a type of

motion. The Resolutions are to be voted upon by the members of the house. Where all

motions are not necessarily put to vote of the House. So, Statement 1 is correct.

A Resolution moved by any member to the resort Amendment, cannot be withdrawn except

by the leave of the house. So, Statement 2 is correct.

Statutory Resolution can be moved by either a private member or the Minister. So,

Statement 3 is not correct.

ADDITIONAL INFORMATION

Types of Resolution

Resolutions are classified into three categories:

• Private Member’s Resolution: It is moved by a private member (other than a minister). It is

discussed only on alternate Fridays and in the afternoon sitting.

• Government Resolution: It is moved by a minister. It can be taken up any day from Monday to

Thursday.

• Statutory Resolution: It can be moved either by a private member or a minister. It is so-called

because it is, always tabled in pursuance of a provision in the Constitution or an Act of

Parliament.

Q.72) Consider the following statements with regard to Financial bill of type II as mentioned

under article 117(3) of the Constitution:

1. A financial bill(II) cannot be passed by either house of the Parliament unless the President has

recommended the consideration of the bill.

2. It can only be introduced in the Lok sabha.

3. There is no provision of joint sitting in case of disagreement between the two houses.

Which of the above statements is/are correct?

(a) 1 and 2 only

(b) 1 only

(c) 2 and 3 only

(d) 1 and 3 only

EXPLANATION

The only special feature of the financial bill (II) is that it cannot be passed by either

House of Parliament unless the President has recommended to that House the consideration

of the bill. So, Statement 1 is correct.

financial bill (II) can be introduced in either House of Parliament and recommendation of the

President is not necessary for its introduction. In other words, the recommendation of the

President is not required at the introduction stage but is required at the Consideration stage.

So, Statement 2 is not correct.

In case of a disagreement between the two Houses over such a bill, the President can

summon a joint sitting of the two Houses to resolve the deadlock. So, Statement 3 is not

correct.

Q.73) Consider the following:

1. National Commission for Scheduled caste(NCSC)

2. National Commission for Backward class(NCBC)

3. Central Vigilance commission(CVC)

4. Special Officer for Linguistic Minorities

Which of the above is/are Constitutional bodies?

(a) 1,3 and 4 only

(b) 1,2 and 3 only

(c) 2,3 and 4 only

(d) 1,2 and 4 only

EXPLANATION

National Commission for Scheduled caste(NCSC) - NCST is directly established by

Article 338 of the Constitution.

National Commission for Backward class(NCBC) - In the Mandal case’ judgement (1992), the Supreme Court directed the central government to constitute a permanent statutory body to examine the complaints of under inclusion, over-inclusion or non-inclusion of any class of citizens in the list of backward classes. Accordingly, the National Commission for Backward Classes (NCBC) was set up in 19937.

Later, the 102nd Amendment Act of 2018 conferred a constitutional status on the

Commission. For this purpose, the amendment inserted a new Article 338-B in the

constitution.

Special Officer for Linguistic Minorities – The 17th Constitutional amendment act of

1956 enacted the Special Officer for Linguistic Minorities based on the recommendations

of the States reorganisation Commission.

Listed above are some of the Constitutional bodies.

Central Vigilance Commission is Non Constitutional body. So, option (d) is correct.

Q.74) Consider the following statements with respect to National Commission for STs:

1. The commission consists of a chairperson, a vice-chairperson along with three other members

who are appointed by the President by warrant under his hand and seal.

2. The recommendations by the commissions on matters related to welfare of Scheduled tribes

are binding on the government.

3. The commission has the power to regulate its own procedure.

Which of the above statements is/are correct?

(a) 1 only

(b) 1 and 3 only

(c) 2 and 3 only

(d) 1,2 and 3

EXPLANATION

The Separate National Commission for STs came into existence in 2004. It consists of a

chairperson, a vice-chairperson and three other members. They are appointed by the President

by warrant under his hand and seal. Their conditions of service and tenure of office are also

determined by the President’. So, Statement 1 is correct.

The recommendations by the commissions on matters related to the welfare of Scheduled tribes

are not binding on the government. So, Statement 2 is not correct.

The Commission is vested with the power to regulate its own procedure. The Commission,

while investigating any matter or inquiring into any complaint, has all the powers of a civil court

trying a suit and in particular in respect of the following matters:

• summoning and enforcing the attendance of any person from any part of India and

examining him on oath;

• requiring the discovery and production of any document;

• receiving evidence on affidavits;

• requisitioning any public record from any court or office;

• issuing summons for the examination of witnesses and documents; and any other matter

which the President may determine

So, Statement 3 is correct.

ADDITIONAL INFORMATION

NCST

The separate National Commission for STs came into existence in 2004 by the 89th Constitutional

Amendment Act 2003. This act further amended Article 338 and inserted a new article 338A in the

Constitution.

Functions

• To investigate and monitor all matters relating to the constitutional and other legal safeguards

for the STS and to evaluate their working;

• To inquire into specific complaints with respect to the deprivation of rights and safeguards of

the STs;

• To participate and advise on the planning process of socio-economic development of the STs

and to evaluate the progress of their development under the Union or a state;

• To present to the President, annually and at such other times as it may deem fit, reports upon

the working of those safeguards;

• To make recommendations as to the measures that should be taken by the Union or a state for

the effective implementation of those safeguards and other measures for the protection, welfare

and socio-economic development of the STs; and

• To discharge such other functions in relation to the protection, welfare and development and

advancement of the STs as the President may specify.

Q.75) With reference to Public Accounts Committee(PAC), consider the following statements:

1. All the members of PAC belong to Lok sabha only.

2. The members are elected by the Parliament every year by method of Proportional

representation.

3. The Chairman is appointed by the speaker of lok sabha and he must belong to the ruling party.

Which of the above statements is/are correct?

(a) 1 and 2 only

(b) 2 only

(c) 2 and 3 only

(d) 1 and 3 only

EXPLANATION

The members of PAC belong to both Lok Sabha and the Rajya Sabha. It consists of 22 members,

15 from Lok Sabha and 7 from Rajya Sabha. So, Statement 1 is not correct.

The members are elected by the Parliament every year from amongst its members according to

the principle of proportional representation by means of the single transferable vote. Thus, all

parties get due representation in it. So, Statement 2 is correct.

The chairman of the committee is appointed from amongst its members by the Speaker. Until

1966 - ‘67, the chairman of the committee belonged to the ruling party. However, since 1967 a

convention has developed whereby the chairman of the committee is selected invariably from the

Opposition. So, Statement 3 is not correct.

Q.76) Consider the following statements with respect to appointment of Supreme court judges:

1. The President can appoint the judge of the Supreme court after consultation with the Chief

justice of India whose recommendations are binding.

2. In the second judge case, the Supreme court held that the Chief justice must constitute a

collegium of four senior-most judges to give recommendations to the President.

Which of the above statements is/are correct?

(a) 1 only

(b) 2 only

(c) Both 1 and 2

(d) Neither 1 nor 2

EXPLANATION

The President can appoint the judge of the Supreme court after consultation with the Chief

justice of India whose recommendations are binding. Where chief justice of India is appointed

by President after consultation with such judges of SV and the HC as he deems necessary. So,

Statement 1 is correct.

In the Second Judges case (1993), the Court reversed its earlier ruling and changed the

meaning of the word consultation to concurrence. Hence, it ruled that the advice tendered by

the Chief Justice of India is binding on the President in the matters of appointment of the

judges of the Supreme Court. But, the Chief Justice would tender his advice on the matter

after consulting two of his senior-most colleagues. So, Statement 2 is not correct.

Q.77) Consider the following statements:

1. The Ad Hoc Judges of the Supreme court are appointed by the President after consultation of

the chief justice of the high court and the Chief justice of India.

2. The Chief justice of India can appoint a retired judge of the Supreme court as judge for a

temporary period only after previous consent of the President and also the person to be

appointed.

Which of the above statements is/are correct?

(a) 1 only

(b) 2 only

(c) Both 1 and 2

(d) Neither 1 nor 2

EXPLANATION

Ad hoc Judge When there is a lack of quorum of the permanent judges to hold or continue any

session of the Supreme Court, the Chief Justice of India can appoint a judge of a High Court

as an ad hoc judge of the Supreme Court for a temporary period. He can do so only after

consultation with the chief justice of the High Court concerned and with the previous consent of

the president. So, Statement 1 is not correct.

At any time, the chief justice of India can request a retired judge of the Supreme Court or a

retired judge of a high court (who is duly qualified for appointment as a judge of the Supreme

Court) to act as a judge of the Supreme Court for a temporary period. He can do so only with

the previous consent of the president and also of the person to be so appointed. So, Statement

2 is correct.

Q.78) With reference to Special Leave Petition, consider the following statements:

1. It is a discretionary power of the Supreme court and High courts and cannot be claimed as a

matter of right.

2. It can be granted against any court or tribunal including a military court.

3. It can only be granted in those judgements where the final award has been given.

Which of the above statements is/are not correct?

(a) 1 only

(b) 2 only

(c) 3 only

(d) 1,2 and 3

EXPLANATION

The Supreme Court is authorized to grant in its discretion special leave to appeal from any

judgement in any matter passed by any court or tribunal in the country (except military

tribunal and court-martial), This provision contains the four aspects as under:

• It is a discretionary power and hence, cannot be claimed as a matter of right.

• It can be granted in any judgement whether final or interlocutory.

• It may be related to any matter—constitutional, civil, criminal, income-tax, labour, revenue,

advocates, etc.

• It can be granted against any court or tribunal and not necessarily against a high

court (of course, except a military court).

Thus, the scope of this provision is very wide and it vests the Supreme Court with a plenary

jurisdiction to hear appeals. On the exercise of this power, the Supreme Court itself held that

‘being an exceptional and overriding power, it has to be exercised sparingly and with caution

and only in special extraordinary situations. Beyond that, it is not possible to fetter the exercise

of this power by any set formula or rule’. So, option (d) is correct.

Q.79) In the Indian context, the concept of Public Interest Litigation(PIL) can be most suitably

derived from?

(a) Judicial adventurism

(b) Judicial review

(c) Judicial activism

(d) Judicial restraint

EXPLANATION

In India, the PIL is a product of the judicial activism role of the Supreme Court. It

was introduced in the early 1980s. Justice V.R. Krishna Iyer and Justice P.N. Bhagwati

were the pioneers of the concept of PIL.

PIL is also known variously as Social Action Litigation (SAL), Social Interest Litigation

(SIL) and Class Action Litigation (CAL).

So, option (c) is correct.

ADDITIONAL INFORMATION

Features of PIL

• PIL is a strategic arm of the legal aid movement and is intended to bring justice within the

reach of the poor masses, who constitute the low visibility area of humanity.

• PIL is a different kind of Litigation from the ordinary traditional litigation which is essential of

an adversary Character where there is a dispute between two litigating parties, one making

claims seeking relief against the other and that other opposing such claim or resisting such

relief.

• PIL demands that violations of constitutional and legal rights of large numbers of people who

are poor, ignorant, or in a socially or economically disadvantaged position should not go

unnoticed and unredressed.

• PIL is essentially a cooperative effort on the part of the petitioner, the State or Public Authority,

and the Court to secure observance of the constitutional or legal rights, benefits and privileges

conferred upon the vulnerable sections of the community and to reach social justice to them.

• In PIL, litigation is undertaken for the purpose of redressing public injury, enforcing public

duty, protecting social, collective, diffused rights and interests or vindicating public interest.

• In PIL, the role held by the Court is more assertive than in traditional actions; it is creative

rather than passive and it assumes a more positive attitude in determining acts.

• Though in PIL court enjoys a degree of flexibility unknown to the trial of traditional private law

litigations, whatever the procedure adopted by the court it must be procedure known to

judicial tenets and characteristics of a judicial proceeding. In a PIL, unlike a traditional dispute

resolution mechanism, there is no determination on the adjudication of individual rights.

Q.80) With respect to languages in India, consider the following statements:

1. The Constitution prescribes Hindi written in devanagari script as the official language of the

Union.

2. The Supreme court hears petitions in both Hindi as well as English language.

3. The President's address at the joint sitting of both the houses of Parliament every year can be

read in both Hindi and English, as preferred by the President.

Which of the above statements is/are correct?

(a) 1 and 2 only

(b) 2 and 3 only

(c) 1 and 3 only

(d) 1,2 and 3

EXPLANATION

Article 343(1) of the Constitution provides that Hindi in Devanagari script shall be the Official

Language of the Union. So, Statement 1 is correct.

Parliament has not made any provision for the use of Hindi in the Supreme Court. Hence SC

hears petitions or appeals only in English. So, Statement 2 is not correct.

The President's address at the joint sitting of both the houses of Parliament every year can be

read in both Hindi or English, as preferred by the President. The other version of Address in

English or Hindi, as the case may be, is read out by the Chairman of Rajya Sabha. So

statement 3 is correct.

Q.81) The origin of Panchayati Raj government as the third tier of government can be best

described by which of the following principles?

1. Democratic Decentralisation

2. Delegated legislation

3. Principle of Subsidiarity

4. Direct Democracy

Select the correct answer using the code given below:

(a) 1 only

(b) 1 and 3 only

(c) 1,2 and 3 only

(d) 1,2,3 and 4

EXPLANATION

Democratic Decentralization is popularly known as Panchayati raj in India. Panchayati raj

institutions have been considered as instruments of socio-economic transformation in rural

India. Decentralization of power to the Panchayats is seen as a means of empowering people

and involving them in decision-making process.

Delighted Legislation – Delegation of power means the act whereby a political authority

invested with certain powers turns over the exercise of those powers, in full or in part, to

another authority. But, in the case of local self-government the opposite happens, where local

bodies exercise their own authority rather than delegating to other organisations.

Principle of Subsidiarity – Subsidiarity holds that such functions of government should be

performed at the lowest level possible, as long as they can be performed adequately. When they

cannot, higher levels of government must intervene. The idea of local self government ensures

this concept.

Direct Democracy – Direct democracy is not a well-suited principle for Panchayati raj, rather

it plays an important role in participatory Democracy. The primary objective of establishing the

third tier of the government is to increase democratic participation, to better articulate local

needs and priorities, and to ensure more efficient use of local resources along with greater

accountability and transparency.

So, option (b) is correct.

Q.82) Consider the following statements:

1. The 73rd Constitutional amendment act makes it mandatory to have a three tier system of

panchayati raj in every state at village, intermediate and district level.

2. While the act explicitly provides for reservation of seats for Scheduled caste, scheduled tribes

and women, it empowers the state legislature to provide reservation to Other backward class in

every panchayat.

Which of the above statements is/are correct?

(a) 1 only

(b) 2 only

(c) Both 1 and 2

(d) Neither 1 nor 2

EXPLANATION

The 73rd Constitutional Amendment Act added Part IX to the Constitution of India. The

provisions of the act can be grouped into two categories—compulsory and voluntary. The

compulsory (mandatory or obligatory) provisions of the act have to be included in the state

laws creating the new Panchayati raj system at the village, intermediate, and district

level.However the states having a population not exceeding 20 lakh may not constitute

panchayats at the intermediate level. So, Statement 1 is not correct.

The act provides for the reservation of seats for scheduled castes and scheduled tribes in

every panchayat (all three levels) in proportion ‘their population to the total population in the

panchayat area. Further, the state legislature shall provide for the reservation of offices of

chairperson in the panchayat tat the village or any other level for the SCs and STs.

The act provides for the reservation of not less than one-third of the total number of feats for

women (including the number of seats reserved for women belonging to the SCs and STs)

The act also authorizes the legislature of a state to make any provision for reservation of

seats in any panchayat or offices of chairperson in the panchayat at any level in favor of

backward classes. So, Statement 2 is correct.

Q.83) The PESA act of 1996 , also known as Extension act, is applicable to?

(a) Both the fifth and sixth schedule areas

(b) Only fifth schedule areas

(c) Only sixth schedule areas

(d) All the North Eastern states

EXPLANATION

PESA act of 1996 is only applicable to the 5th schedule areas.

The provisions of Part IX of the constitution relating to the Panchayats do not apply to the Fifth

Schedule areas. However, the Parliament may extend these provisions to such areas, subject to

such exceptions and modifications as it may specify. Under this provision, the Parliament has

enacted the “Provisions of the Panchayats (Extension to the Scheduled Areas) Act”, 1996,

popularly known as the PESA Act or the Extension Act.

At present (2019), ten states have Fifth Schedule Areas. These are Andhra Pradesh, Telangana,

Chhatisgarh, Gujarat, Himachal Pradesh, Jharkhand, Madhya Pradesh, Maharashtra, Odisha

and Rajasthan. All the ten states have enacted requisite compliance legislation by amending

the respective Panchayati Raj Acts.

So, option (b) is correct.

ADDITIONAL INFORMATION

Features of PESA Act

• A State legislation on the Panchayats in the Scheduled Areas shall be in consonance with the

customary law, social and religious practices and traditional management practices of

community resources.

• A village shall ordinarily consist of a habitation or a group of habitations or a hamlet or a

group of hamlets comprising a community and managing its affairs in accordance with

traditions and customs.

• Every village shall have a Gram Sabha consisting of persons whose names are included in the

electoral rolls for the Panchayat at the village level.

• Every Gram Sabha shall be competent to safeguard and preserve the traditions and customs of

the people, their cultural identity, community resources and the customary mode of dispute

resolution.

• Every Gram Sabha shall

I. approve of the plans, programs and projects for social and economic development

before they are taken up for implementation by the Panchayat at the village level; and

II. be responsible for the identification of beneficiaries under the poverty alleviation and

other programmes.

• Every Panchayat at the village level shall be required to obtain from the Gram Sabha a

certification of utilisation of funds for the above plans, programmes and projects.

• The reservation of seats in the Scheduled Areas in every Panchayat shall be in proportion to

the population of the communities for whom reservation is sought to be given under Part IX of

the Constitution. However, the reservation for the Scheduled Tribes shall not be less than one-

half of the total number of seats. Further, all seats of Chairpersons of Panchayats at all levels

shall be reserved for the Scheduled Tribes.

• The state government may nominate such Scheduled Tribes which have no representation in

the Panchayat at the intermediate level or the Panchayat at the district level. But such

nomination shall not exceed one-tenth of the total members to be elected in that Panchayat.

• The Gram Sabha or the Panchayats at the appropriate level shall be consulted before making

the acquisition of land in the Scheduled Areas for development projects and before resettling or

rehabilitating persons affected by such projects in the Scheduled Areas. However, the actual

planning and implementation of the projects in the Scheduled Areas shall be coordinated at

the state level.

• Planning and management of minor ¢ ‘water bodies in the Scheduled Areas shall be entrusted

to Panchayats at the? appropriate level.

• The recommendations of the Gram Sabha or the Panchayats at the appropriate level shall be

mandatory for grant of prospecting license or mining lease for minor minerals in the

Scheduled Areas.

Q.84) Which of the following offices is/are appointed by the Governor of a state?

1. State Election Commission(SEC)

2. State Finance Commission

3. Accountant General

4. Judges of high court

Select the correct answer using the code given below:

(a) 1,2 and 3 only

(b) 1 and 2 only

(c) 1,3 and 4 only

(d) 2 and 3 only

EXPLANATION

Listed here are some of the offices appointed by Governor of state

1. Advocate General

2. Chief Minister

3. State Election Commissioners

4. Chairman and members of Public Service Commission

So, option (b) is correct.

Q.85) With reference to evolution of Urban local bodies in India, which of the following

statements is/are correct?

1. The first municipal corporation in India was set up in the 17th century in Bombay province.

2. Lord Ripon’s resolution of 1882 is called the Magna Carta of local self government in India.

3. The British colonial laws mainly focussed on development of urban local bodies with negligible

attention to rural areas.

Select the correct answer using the code given below:

(a) 1 and 2 only

(b) 2 and 3 only

(c) 1 and 3 only

(d) 1,2 and 3

EXPLANATION

The first municipal corporation was set up in the 16th century in Madras. So, Statement 1 is

not correct.

Lord Ripon’s resolution of 1882 is called the Magna Carta of local self-government in India. He

is called the father of local self-government in India. So, Statement 2 is correct.

British colonial laws are mainly focused on the urban local bodies because of the availability of

resources and urban cities like Madras and Kolkata have ports which are used for trading so it

is necessary for the British to concentrate in urban local bodies with negligible attention to

rural areas. Rapid urbanization in the state over the past few decades is witnessed in the

growth of municipal bodies. The numbers of municipal towns have gone up from 93 in 1951 to

122 in 2000. So, Statement 3 is correct.

Q.86) Which of the following can be represented in a municipality as per the 74th

Constitutional amendment act?

1. Persons with special knowledge or experience in municipal administration.

2. Members of Lok sabha and Rajya sabha within the municipal area

3. Members of the state legislative assembly.

Select the correct answer using the code given below:

(a) 1 only

(b) 2 and 3 only

(c) 1 and 3 only

(d) 1,2 and 3

EXPLANATION

All members of a municipality shall be elected directly by the people of the municipal area, For

this purpose, each won municipal area shall be divided into territorial constituencies to be

known as wards. The state legislature may provide the manner of in of election of the

chairperson of a municipality. It may also provide for the representation of the following persons

in a municipality.

• Persons having special knowledge or experience in municipal administration without the

right to vote in the meetings of municipality.

• The members of the Lok Sabha and the state legislative assembly representing constituencies

that comprise wholly or partly the municipal area.

• The members of the Rajya Sabha and the state legislative council registered as electors

within the municipal area.

• The chairpersons of committees (other than wards committees).

So, option (d) is correct.

Q.87) Which of the following is an entirely elected body?

1. District Planning committee

2. Metropolitan planning committee

3. Notified Area committee

Select the correct answer using the code given below:

(a) 1 and 2 only

(b) 3 only

(c) 1,2 and 3

(d) None of these

EXPLANATION

District Planning committee – The 4/5th of the members should be elected by the elected

members of district panchayat and municipalities in the district among themselves. Here 1/5

members are nominated.

Metropolitan planning committee - The act lays down that two-thirds of the members of a

metropolitan planning committee should be elected by the elected members of the municipalities

and chairpersons of the panchayats in the metropolitan area from amongst themselves. The

representation of these members in the committee should be in proportion to the ratio between

the population of the municipalities and the panchayats in that metropolitan area. Here 1/3rd

members are nominated.

Notified Area committee - It is an entirely nominated body, that is, all the members of a

notified area committee including the chairman are nominated by the state government. Thus, it

is neither an elected body nor a statutory body.

So, option (d) is correct.

ADDITIONAL INFORMATION

District Planning committee

• Every state shall constitute at the district level, a district planning committee to Consolidate

the plans prepared by panchayats and municipalities the in the district , and to prepare a

draft development plan for the district as a whole.

• The state legislature may make provisions with respect to the following:

1. The composition of such committees;

2. The manner of election of members of such committees;

3. The functions of such committees in relation to district planning; and

4. The manner of the election of the chair_ Persons of such committees.

Metropolitan planning committee

• Every metropolitan area shall have a metropolitan planning committee to prepare a draft

development plan’. The state legislature may make provisions with respect to the following:

1. The composition of such committees;

2. The manner of election of members of such committees;

3. The representation in such committees of the Central government, state government

and other organisations;

4. The functions of such committees in relation to planning and coordination for the

metropolitan area; and

5. The manner of election of chairpersons of such committees.

Notified Area committee

• A notified area committee is created for the administration of two types of areas—a fast

developing town due to industrialisation, and a town which does not yet fulfil all the conditions

necessary for the constitution of a municipality, but which otherwise is considered important

by the state government.

• Since it is established by notification in the government gazette, it is called as notified area

committee.

• Though it functions within the framework of the State Municipal Act, only those provisions of

the act apply to it which are notified in the government gazette by which it is created. It may

also be entrusted to exercise powers under any other act.

• Its powers are almost equivalent to those of a municipality.

Q.88) Consider the following statements with respect to Union Public Service

Commission(UPSC):

1. The Process of removal of the Chairman or members of UPSC is same as the members of

Election commission of India.

2. The Chairman as well as members of UPSC are not eligible for further employment of the

Government of India or the state.

3. The jurisdiction of UPSC can be extended by an act made by the Parliament.

Which of the above statements is/are correct?

(a) 1 and 2 only

(b) 1 and 3 only

(c) 3 only

(d) 2 and 3 only

EXPLANATION

The Process of removal of the Chairman or members of UPSC is not same as the members of the

Election commission of India.

The President can remove the chairman or any other member of UPSC from the office under the following circumstances:

(a) If he is adjudged an insolvent (that is, has gone bankrupt);

(b) If he engages, during his term of office, in any paid employment outside the duties of his office; or

(c) If he is, in the opinion of the president, unfit to continue in office by reason of infirmity of

mind or body.

President can also remove the chairman or any other member of UPSC for misbehavior.

However, in this case, the president has to refer the matter to the Supreme Court for an

enquiry. If the Supreme Court, after the enquiry, upholds the cause of removal and advises so,

the president can remove the chairman or a member. Under the provisions of the Constitution,

the advice tendered by the Supreme Court in this regard is binding on the President.

Where ECI is removed as the Supreme Court judge is removed. He can be removed by the

president on the basis of a resolution passed to that effect by both the Houses of Parliament

with a special majority, either on the ground of proved misbehaviour or incapacity. So,

Statement 1 is not correct.

The chairman of UPSC is not eligible for further employment of the Government of India or the

state. However, members of UPSC are eligible for further appointment as the chairman of UPSC

or SPSC but not for any other employment in the GOI and state. So, Statement 2 is not

correct.

The additional functions relating to the services of the Union can be conferred on UPSC by the

Parliament. It can also place the personnel system of any authority, corporate body, or public

institution within the jurisdiction of the UPSC. Hence the jurisdiction of UPSC can be extended

by an act made by the Parliament. So, Statement 3 is correct.

ADDITIONAL INFORMATION

• UPSC is the central recruiting agency in India. It is an independent Constitutional body in the

sense that it has peen directly created by the Constitution.

• Articles 315 to 323 in Part XIV of the Constitution contain elaborate provisions regarding the

composition, appointment and removal of members along with the independence, powers and

functions of the UPSC.

• The UPSC consists of a chairman and other members appointed by the president of India. The

Constitution, without specifying the strength of the Commission has left the matter to the

discretion of the president, who determines its composition.

• Usually, the Commission consists of nine to eleven members including the chairman. Further,

no qualifications are prescribed for the Commission's membership except that one-half of the

members of the Commission should be such persons who have held office for at least ten years

either under the Government of India or under the government of a state.

• The Constitution also authorises the president to determine the conditions of service of the

chairman and other members of the Commission.

Q.89) With reference to CAG of India, consider the following statements:

1. CAG compiles and maintains state governments but not of the central government.

2. The scope of audit of CAG with respect to expenditure of the government is more than that of

audit of receipt of the government.

3. The CAG is responsible only to the Parliament of India.

Which of the above statements is/are correct?

(a) 1 and 2 only

(b) 2 and 3 only

(c) 1 and 3 only

(d) 1,2 and 3

EXPLANATION

CAG compiles and maintains only for the State Governments. The Parliament enacted the

CAGs (Duties, Powers and Conditions of service) act in 1971 and was amended in 1976 to

separate accounts from audit in the Central Government. After this amendment CAG was

relived from the duty of maintaining accounts of the Central government. So, Statement 1 is

correct.

The CAG has more freedom with regard to the audit of expenditure than with regard to the

audit of receipts, stores, and stock. “Whereas in relation to expenditure he decides the scope

of the audit and frames his own audit codes and manuals, he has to proceed with the

approval of the executive government in relation to rules for the conduct of the other audits.

So, Statement 2 is correct.

The role of CAG is to uphold the Constitution of India and the laws of Parliament in the field

of financial administration. The CAG is an agent of the Parliament and conducts an audit of

expenditure on behalf of the Parliament. Therefore, he is responsible only to the Parliament.

So, Statement 3 is correct.

ADDITIONAL INFORMATION

CAG

• The Constitution of India provides for an independent office of the Comptroller and Auditor

General of India (CAG). He is the head of the Indian Audit and Accounts Department’.

• He is the guardian of the public purse and controls the entire financial system of the country

at both the levels—the Centre and the state.

• His duty is to uphold the Constitution of India and laws of Parliament in the field of financial

administration.

• CAG audits the accounts related to all expenditure from the Consolidated Fund of India,

consolidated fund of each state and consolidated fund of each union territory having a

Legislative Assembly.

• He audits all expenditures from the Contingency Fund of India and the Public Account of India

as well as the contingency fund of each state and the public account of each state.

• He audits all trading, manufacturing, profit and loss accounts, balance sheets and other

subsidiary accounts kept by any department of the Central Government and state

governments.

Q.90) Consider the following statements:

1. A community can be recognised as a scheduled tribe if it qualifies the criteria laid down in the

Constitution.

2. Any inclusion or exclusion of a tribe from the list of scheduled tribes can only be done through

Presidential notification.

Which of the above statements is/are correct?

(a) 1 only

(b) 2 only

(c) Both 1 and 2

(d) Neither 1 nor 2

EXPLANATION

Constitution does not have any criteria to be recognize a community as the Scheduled tribes. So,

Statement 1 is not correct.

Parliament may be law include in or exclude from the list of Scheduled Tribes or part of or group

within any tribe or tribal community. President may with respect to any State or Union Territory,

and where it is a State, after consultation with the Governor thereof, by public notification, specify

the tribes or tribal communities or part of or groups within tribes or tribal communities as

Scheduled Tribe in relation to that State or Union Territory. So, Statement 2 is not correct.

Q.91) Which of the following are the members of the selection committee formed for the

selection of chairperson and members of Lokpal?

1. Prime Minister

2. Eminent jurist nominated by the President of India

3. Leader of opposition in lok sabha

4. Speaker of Lok sabha

5. Chief justice of India

Select the correct answer using the code given below:

(a) 1,4 and 5 only

(b) 1,2,4 and 5 only

(c) 1,2 and 3 only

(d) 1,2,3,4 and 5

EXPLANATION

The members are appointed by the president on the recommendation of a Selection Committee.

The selection committee is composed of the Prime Minister who is the Chairperson; Speaker of

Lok Sabha, Leader of Opposition in Lok Sabha, Chief Justice of India or a Judge nominated by

him/her, and One eminent jurist. So, option (d) is correct.

ADDITIONAL INFORMATION

Lok Pal

• The Lokpal is the first institution of its kind in independent India, established under the

Lokpal and Lokayuktas Act 2013 to inquire and investigate allegations of corruption against

public functionaries who fall within the scope and ambit of the above Act.

• The Lokpal of India is committed to addressing the concerns and aspirations of the citizens of

India for clean governance.

• It shall make all efforts within its jurisdiction to serve the public interest and shall endeavor to

use the powers vested in it to eradicate corruption in public life.

• India is a signatory to the United Nations Convention against Corruption.

• The commitment of the Government to provide clean and responsive governance is reflected in

the passing of the legislation and creation of the body of Lokpal, to contain and punish acts of

corruption.

• The Lokpal is vested with the power of search and seizure and also powers under the Civil

Procedure Code for the purpose of conducting preliminary inquiry & investigation and power of

attachment of assets and taking other steps for eradication of corruption.

• Lokpal will have the power of superintendence and direction over any central investigation

agency including CBI for cases referred to them by the Lokpal.

Q.92) Which of the following Constitutional amendment acts gave Constitutional status to the

Co-operative societies in India?

(a) 97th Constitutional amendment act

(b) 95th Constitutional amendment act

(c) 85th Constitutional amendment act

(d) 86th Constitutional amendment act

EXPLANATION

97th Constitutional amendment Act gave Constitutional status to the Co-operative societies in

India. So, option (a) is correct.

ADDITIONAL INFORMATION

Laws governing Cooperative Societies

• Cooperation is in the concurrent list, which means both the central and state governments can

govern them.

• A majority of the cooperative societies are governed by laws in their respective states, with a

Cooperation Commissioner and the Registrar of Societies as their governing office.

• In 2002, the Centre passed a Multi-State Cooperative Societies Act that allowed for the

registration of societies with operations in more than one state.

• These are mostly banks, dairies, and sugar mills whose area of operation spreads across

states.

• The Central Registrar of Societies is their controlling authority, but on the ground, the State

Registrar takes action on his behalf.

Constitutional Provisions Related to Cooperatives

• The 97th Amendment Constitutional Act, 2011 added a new Part IXB regarding the

cooperatives working in India.

• This Act amended Article 19(1)(c) of the Constitution by adding the word “cooperatives” after

“unions and associations”. This enables all the citizens the fundamental right to form

cooperatives.

• A new Article 43B was added in the Directive Principles of State Policy regarding the

“promotion of cooperative societies”.

Q.93) Consider the following statements with regard to the Central Administrative tribunal

created under Article 323 A of the Constitution:

1. The CAT exercises original jurisdiction in relation to service matters of members of all India

services, central civil service, defence forces and servants of the Supreme court.

2. The appeals against the order of CAT can be made only in the Supreme court.

3. It is guided by the principle of natural justice and is not bound by the civil procedure code of

1908.

Which of the above statements is/are correct?

(a) 1 and 3 only

(b) 3 only

(c) 2 and 3 only

(d) 1 and 2 only

EXPLANATION

The CAT exercises original jurisdiction in relation to recruitment and all service matters of

public servants covered by it. Its jurisdiction extends to the all-India services, the Central civil

services, civil posts under the Centre, and civilian employees of defense services. However, the

members of the defense forces, officers and servants of the Supreme court, and the secretarial

staff of the Parliament are not covered by it. So, Statement 1 is not correct.

Originally, appeals against the orders of the CAT could be made only in the Supreme Court

and not in the high courts. However, in the Chandra Kumar case” (1997), the Supreme Court

declared this restriction on the jurisdiction of the high courts as unconstitutional, holding

that judicial review is a part of the basic structure of the Constitution. It laid down that

appeals against the orders of the CAT shall lie before the division bench of the concerned high

court. Consequently, now it is not possible for an aggrieved public servant to approach the

Supreme Court directly against an order of the CAT, without first going to the concerned high

court. So, Statement is not correct.

The CAT is not bound by the procedure laid down in the Civil Procedure Code of 1908. It is

not guided by the principles of natural justice. These principles keep the CAT flexible in

approach. Only a nominal fee of Rs50 is to be paid by the applicant. So, Statement 3 is

correct.

ADDITIONAL INFORMATION

CAT

• The 42nd Amendment Act of 1976 added a new Part XIV-A to the Constitution This part is

entitled as ‘Tribunals’ and consists of only two Articles—Article 323 A dealing with

administrative tribunals and Article 323 B dealing with tribunals for other matters.

• Article 323 A empowers the Parliament to provide for the establishment of administrative

tribunals for the adjudication of disputes relating to recruitment and conditions of service of

persons appointed to public services of the Centre, the states, local bodies, public corporations

and other public authorities.

• In other words, Article 323 A enables the Parliament to take out the adjudication of disputes

relating to service matters from the civil courts and the high courts and place it before the

administrative tribunals.

• Under Article 323 B, the Parliament and the state legislatures are authorized to provide for the

establishment of tribunals for the adjudication of disputes relating to the fol. lowing matters:

1. Taxation

2. Foreign exchange, import and export (c) Industrial and labor

3. Land reforms

4. Ceiling on urban property

5. Elections to Parliament and state legislatures

6. Foodstuffs

7. Rent and tenancy rights

• The CAT is a multi-member body consisting of a chairman and members. Originally, the CAT

consisted of a Chairman, Vice-Chairman, and members. Later, in 2006, the provision for the

Vice-Chairman was removed by the Administrative Tribunals (Amendment) Act, 2006.

• Hence, there is now no Vice-Chairman in the CAT. At present (2019), the sanctioned strength

of the chairman is one, and sanctioned strength of Members is 65.

• They are drawn from both judicial and administrative streams and are appointed- by -the

President.

• They hold office for a term of five years or until they attain the age of 65 years in the case of

chairman and 62 years in case of members, whichever is earlier.

Q.94) Consider the following statements with respect to Model Code of Conduct(MCC) in

context of Indian elections:

1. MCC was evolved by the Parliament of India on the basis of consensus among the political

parties in 1968.

2. MCC comes into effect from the day on which election schedules are announced by the

Election commission of India.

3. It is not enforceable by law.

Which of the above statements is/are correct?

(a) 1 and 2 only

(b) 2 and 3 only

(c) 3 only

(d) 1,2 and 3

EXPLANATION

In 1968, the Election Commission held meetings with political parties at the State level and

circulated the Code of Conduct to observe the minimum standard of behavior to ensure free

and fair elections. It was not evolved by the Parliament. So, Statement 1 is not correct.

MCC comes into operation from the day ECI announces the program for election. It ensures

that electoral offences, malpractices, and corrupt practices such as impersonation, bribing

and inducement of voters, threat and intimidation to the voters, are prevented by all

means. So, Statement 2 is correct.

The MCC is not enforceable by law. However, certain provisions of the MCC may be

enforced through invoking corresponding provisions in other statutes such as the Indian

Penal Code, 1860, Code of Criminal Procedure, 1973, and Representation of the People Act,

1951. So, Statement 3 is correct

ADDITIONAL INFORMATION

MCC

The Model Code of Conduct (MCC) is a set of guidelines issued by the Election Commission of India for

the conduct of political parties & candidates during elections. The norms have been evolved with the

consensus of political parties who have consented to abide by the principles embodies in the said code

and also binds them to respect and observe it in its letter and spirit.

Provisions

• No Minister either of Central or State government shall undertake an official visit of any

constituency from which elections have been announced by ECI till the end of the election

process.

• All arrangements for the campaign purposes are to be made by non-officials and not by

government servants except those relating to the maintenance of law and order.

• Under no circumstances, political or personal visits of the Ministers can be combined with

official visits during MCC.

• Ministers traveling for official work should not halt in the constituency where MCC is in place.

• Any entertainment at state cost on a religious occasion is prohibited.

• The contesting candidates and their campaigners must respect the home life of others . The

right of every individual for peaceful and undisturbed home life should be fully safeguarded

• Criticism of other political parties, when made, shall be confined to their policies and

programme, past record and work. Parties and Candidates shall refrain from criticism of all

aspects of private life

• The public spaces like meeting grounds, helipads, should be equally shared among the

contesting candidates . These public spaces should not be allowed to be monopolized by a few

candidates.

• No party or candidate shall include in any activity which may aggravate existing differences or

create mutual hatred or cause tension between different castes and communities, religious or

linguistic.

Q.95) With reference to election process, consider the following statements:

1. The nominated candidates for election can withdraw their nomination within two days from the

date of scrutiny of their nomination papers.

2. The candidate must be present in physical form before the returning officer to subscribe the

oath or affirmation.

3. The official campaign of candidates must end 49 hours before the polling closes in that

constituency.

Which of the above statements is/are not correct?

(a) 1 only

(b) 2 only

(c) 3 only

(d) None of these

EXPLANATION

1. The formal process for the elections starts with the Notification or Notifications calling upon

the electorate to elect Members of a House. As soon as Notifications are issued, candidates

can start filing their nominations in the constituencies from where they wish to contest.

These are scrutinized by the Returning Officer of the constituency concerned after the last

date for the same is over after about a week. The validly nominated candidates can

withdraw from the contest within two days from the date of scrutiny. Contesting

candidates get at least two weeks for a political campaign before the actual date of the poll.

So, Statement 1 is correct.

2. A candidate must make and subscribe to an oath or affirmation before an officer authorized

by the Election Commission. For any particular election, the authorized persons are,

principally, the Returning Officer and the Assistant Returning Officer for the constituency.

In the case of a candidate confined in a prison or under preventive detention, the

superintendent of the prison or commandant of the detention camp in which he is so

confined or is under such detention is authorized to administer the oath. And in the case of

a candidate confined to bed in a hospital or elsewhere owing to illness or any other cause,

the medical superintendent in charge of the hospital or the medical practitioner attending

on him is similarly authorized. If a candidate is outside India, the Indian Ambassador or

High Commissioner or diplomatic consular authorized by him can also administer

oath/affirmation. The candidate, in person, is required to take the oath or affirmation

immediately after presenting his nomination papers and in any case not later than the day

previous to the date of the scrutiny’. So, Statement 2 is not correct.

3. The campaign is the period when the political parties put forward their candidates and

arguments with which they hope to persuade people to vote for their candidates and

parties. Candidates are given a week to put forward their nominations. These are

scrutinized by the Returning Officers and if not found to be in order can be rejected after a

summary hearing. Validly nominated candidates can withdraw within two days after

nominations have been scrutinized. The official campaign lasts at least two weeks from

the drawing up of the list of nominated candidates, and officially ends 48 hours

before polling closes. So, Statement 3 is not correct

All the given statements are correct. So, option is correct.

Q.96) Consider the following statements:

1. Only Parliament and not the state legislature is empowered to make laws on all matters

relating to elections to the Parliament and the state legislatures.

2. The orders issued by the delimitation commission relating to delimitation of constituencies

cannot be challenged in any court.

3. Disputes related to elections can only be appealed in the Supreme court of India.

Which of the above statements is/are correct?

(a) 1 and 2 only

(b) 2 and 3 only

(c) 2 only

(d) 1 and 3 only

EXPLANATION

Parliament may make provisions concerning all matters relating to elections to the Parliament

and the state legislatures including the preparation of electoral rolls, the delimitation of

constituencies, and all other matters necessary for securing their due constitution. The state

legislatures can also make provision with respect to all matters relating to elections to the

state legislatures including the preparation of electoral rolls and all other matters

necessary for securing their due constitution. But, they can make provision for only those

matters which are not covered by the Parliament. In other words, they can only supplement the

parliamentary law and cannot override it. So, Statement 1 is not correct.

The Constitution declares that the validity of any law relating to the delimitation of

constituencies or the allotment of seats to such constituencies cannot be questioned in any

court. Consequently, the orders issued by the Delimitation Commission become final and cannot

be challenged in any court. So, Statement 2 is correct.

Disputes related to elections can only be appealed in the High court of India, not by the Supreme

Court. So, Statement 3 is not correct.

Q.97) Consider the following:

1. Allocation of seats in the lok sabha and state legislature

2. Preparation of electoral rolls for Parliamentary constituencies.

3. Notification of general elections

4. Registration of political parties.

Which of the above subject matters related to election are contained in Representation of People

act, 1951?

(a) 2,3 and 4 only

(b) 1,3 and 4 only

(c) 1 and 2 only

(d) 3 and 4 only

EXPLANATION

Subject matters related to the election are contained in the Representation of People act,

1951.

1. Qualifications and disqualifications for membership of Parliament and State Legislatures

2. Notification of general elections

3. Administrative machinery for the conduct of elections

4. Registration of political parties

5. Conduct of elections

6. Free supply of certain material to candidates of recognized political parties

7. Disputes regarding elections

8. Corrupt practices and electoral offenses

9. Powers of Election Commission in connection with inquiries as to disqualifications of

members.

10. Bye-elections and time limit for filling vacancies.

11. Miscellaneous provisions relating to elections.

12. Barring the jurisdiction of civil courts.

So, option (d) is correct.

Q.98) Consider the following:

1. Judicial review

2. Division of power

3. Separation of power

Which of the above do not limit the sovereignty of Indian Parliament?

(a) 1 and 2 only

(b) 3 only

(c) 2 only

(d) None of these

EXPLANATION

Judicial Review - Judicial review can be struck down by the laws made by the Parliament

if it is found to be violative to the Constitution. Interfere by the Supreme Court limit the

Parliament in making laws that limit the sovereignty of the Indian Parliament.

Division of powers - This leads to Division of powers between the Central Government and

the state governments. In ordinary circumstances the Central Government cannot legislate

on the matters mentioned in the state this limits the sovereignty of the Indian Parliament.

Separation of Powers - Separation of powers divides the mechanism of governance into three branches i.e. Legislature, Executive, and the Judiciary. Parliament doesn’t have the power to make laws that are violative. The judiciary can strike down laws that it considers unconstitutional or arbitrary. The legislature, on its part, has protested against judicial activism and tried to frame laws to circumvent certain judgments. Judicial activism is said to be against the principle of separation of powers. There have been instances where the courts have issued laws and policies through judgements. For example, the Vishakha Guidelines where the SC issued guidelines on sexual harassment. Thus Division of power limits the sovereignty of Indian Parliament.

All three limits the sovereignty of Indian Parliament. So, option (d) is correct

Q.99) Consider the following statements with respect to Anti defection law in India:

1. It was added in the Constitution by the 52nd constitutional amendment act of 1985 and later

amended by 91st amendment act of 2003.

2. It provides for defection as a ground for disqualification for the elected members of the house

only.

3. The presiding officer of the house is the final decision authority under the law and his decision

cannot be challenged in a court of law.

Which of the above statements is/are correct?

(a) 1 only

(b) 1 and 2 only

(c) 2 and 3 only

(d) 1,2 and 3

EXPLANATION

The 52nd Amendment Act of 1985 provided for the disqualification of the members of

Parliament and the state legislatures on the ground of defection from one political party to

another. For this purpose, it made changes in four Articles’ of the Constitution and added a

new Schedule (the Tenth Schedule) to the Constitution. This act is often referred to as the

‘anti-defection law’. Later, the 91% Amendment Act of 2003 made one change in the

provisions of the Tenth Schedule. It omitted an exception provision i.e., disqualification on the

ground of defection not to apply in case of split. So, Statement 1 is correct.

It provides for defection as a ground for disqualification for all members elected members

including the independent and the nominated members of the house. So, Statement 2 is not

correct.

Any question regarding disqualification arising out of defection is to be decided by the

presiding officer of the House. Originally, the act provided that the decision of the presiding

officer is final and cannot be questioned in any court. However, in the Kihoto Hollohan case

(1993), the Supreme Court declared this provision as unconstitutional on the ground that it

seeks to take away the jurisdiction of the Supreme Court and the high courts. It held that

the presiding officer while deciding a question under the Tenth Schedule, functions as a

tribunal. Hence, his decision like that of any other tribunal is subject to judicial review

on the grounds of malafides, perversity, etc. So, Statement 3 is not correct.

ADDITIONAL INFORMATION

Disqualifications

Members of Political Parties: A member of a House belonging to any political party becomes

disqualified for being a member of the House,

a. if he voluntarily gives up his membership of such political party; or

b. if he votes or abstains from voting in such House contrary to any direction issued by his

political party without obtaining prior permission of such party and such act has not been

condoned by the party within 15 days.

From the above provision it is clear that a member elected on a party ticket should continue in the

party and obey the party directions.

Independent Members: An independent member of a House (elected without being set up as a

candidate by any political party) becomes disqualified to remain a member of the House if he joins any

political party after such election.

Nominated Members: A nominated member of a House becomes disqualified for being a member of

the House if he joins any political party after the expiry of six months from the date on which he takes

his seat in the House.

This means that he may join any political party within six months of taking his seat in the House

without inviting this disqualification.

Q.100) Consider the following statements with respect to powers of the Election commission of

India(ECI):

1. The ECI has the power to recognise as well as derecognise political parties in India.

2. ECI can cancel polls in the event of rigging , booth capturing , violence or other irregularities as

it deems fit.

3. ECI advises the President and Governors on matters relating to disqualification of members of

Parliament and state legislature respectively.

Which of the above statements is/are correct?

(a) 1 and 2 only

(b) 2 and 3 only

(c) 1 and 3 only

(d) 1,2 and 3

EXPLANATION

The ECI has only powers to recognize political parties, it does not have the power to

derecognize the political party. So, Statement 1 is not correct.

ECI has the power to cancel the polls in event of rigging, booth capturing, violence, and other

irregularities. So, Statement 2 is correct.

ECI advises the President and Governors on matters relating to disqualification of members of

Parliament and state legislature respectively. The President is bound by the Commission’s

recommendations. So, Statement 3 is correct.

ADDITIONAL INFORMATION

ECI

• The Election Commission is a permanent and independent body established by the

Constitution of India directly to ensure free and fair elections in the country.

• Article 324 of the Constitution provides that the power of superintendence, direction, and

control of elections to parliament, state legislatures, the office of president of India, and the

office of vice-president of India shall be vested in the election commission.

• Thus, the Election Commission is an all-India body in the sense that it is common to both the

Central government and the state governments.

• It must be noted here that the election commission is not concerned with the elections to

panchayats and municipalities in the states. For this, the Constitution of India provides for a

separate State Election Commission.

• Election Commission has been functioning as a multi-member body consisting of three election

commissioners.

COMPOSITION

Article 324 of the Constitution has made the following provisions about the composition of the election

commission:

• The Election Commission shall consist of the chief election commissioner and the such

number of other election commissioners, if any, as the president may from time to time fix.

• The appointment of the chief election commissioner and other election commissioners shall be

made by the president.

• When any other election commissioner is so appointed, the chief election commissioner shall

act as the chairman of the election commission.

• The president may also appoint after consultation with the election commission such as

regional commissioners as he may consider necessary to assist the election commission.

• The conditions of service and tenure of office of the election commissioners and the regional

commissioners shall be determined by the president


Recommended